Закон ома треугольник: Закон Ома для участка цепи – формула, определение сопротивления

Содержание

простое объяснение для чайников с формулой и понятиями. Закон Ома для электрической цепи

Автор Andrey Ku На чтение 31 мин Опубликовано

Содержание

  1. Закон Ома для цепи
  2. Для ЭДС
  3. Для полной цепи
  4. Для переменного тока
  5. Для замкнутой цепи
  6. Сфера применения
  7. Закон Ома для замкнутой цепи
  8. Закон Ома для неоднородного участка цепи
  9. Закон Ома для переменного тока
  10. Метода треугольника закона Ома
  11. Значение Закона Ома
  12. Сопротивление
  13. Таблица удельных сопротивлений различных материалов
  14. Резистор
  15. Реостат
  16. Закон Ома для участка цепи
  17. Закон Ома для полной цепи
  18. Что такое ЭДС и откуда она берется
  19. Когда «сопротивление бесполезно»
  20. Формула Закона Джоуля-Ленца
  21. Преобразованные формулы Закона Ома и Джоуля-Ленца
  22. Применение закона Ома на практике
  23. Таблица-шпаргалка
  24. Последовательное и параллельное включение элементов
  25. Цепь последовательно включенных резистивных элементов
  26. Цепь параллельно включенных резистивных элементов
  27. Интегральная и дифференциальная формы закона
  28. Параллельное и последовательное соединение
  29. Последовательное соединение
  30. Параллельное соединение
  31. Что нам дает параллельное и последовательное соединение?
  32. Интегральная и дифференциальная формы закона

Закон Ома для цепи

Закон Ома для участка цепи, безусловно, можно описать известной из школьного курса физики формулой: I=U/R, но некоторые изменения и уточнения внести, думаю, стоит.  Возьмем замкнутую электрическую цепь и рассмотрим ее участок между точками 1-2. Для простоты я взял участок электрической цепи, не содержащий источников ЭДС (Е).

Итак, закон Ома для рассматриваемого участка цепи имеет вид:

φ1-φ2=I*R, где

  • I – ток, протекающий по участку цепи.
  • R – сопротивление этого участка.
  • φ1-φ2 – разность потенциалов между точками 1-2.

Если учесть, что разность потенциалов это напряжение, то приходим к производной формулы закона Ома, которая приведена в начале страницы: U=I*R. Это формула закона Ома для пассивного участка цепи (не содержащего источников электроэнергии).

Интересно по теме: Как проверить стабилитрон.

В неразветвленной электрической цепи (рис.2) сила тока во всех участках одинакова, а напряжение на любом участке определяется его сопротивлением:

  • U1=I*R1
  • U2=I*R2
  • Un=I*Rn
  • U=I*(R1+R2+…+Rn

Отсюда можно получить формулы, которые пригодятся при практических вычислениях. Например:

U=U1+U2+…+Un или U1/U2/…/Un=R1/R2/…/Rn

Расчет сложных (разветвленных) цепей осуществляется с помощью законов Кирхгофа.


Закон Ома для участка цепи.

Для ЭДС

Перед тем как рассмотреть закон Ома для полной (замкнутой) цепи приведу правило знаков для ЭДС, которое гласит:


Если внутри источника ЭДС ток идет от катода (-) к аноду (+) (направление напряженности поля сторонних сил совпадает с направлением тока в цепи, то ЭДС такого источника считается положительной. В противном случае – ЭДС считается отрицательной.

Практическим применением этого правила является возможность приведения нескольких источников ЭДС в цепи к одному с величиной E=E1+E2+…+En, естественно, с учетом знаков, определяемых по вышеприведенному правилу. Например (рис.3.3) E=E1+E2-E3. При отсутствии встречно включенного источника E3 (на практике так почти никогда не бывает) имеем широко распространенное последовательное включение элементов питания, при котором их напряжения суммируются.

Для полной цепи

Закон Ома для полной цепи – его еще можно назвать закон ома для замкнутой цепи, имеет вид I=E/(R+r). Приведенная формула закона Ома содержит обозначение r, которое еще не упоминалось. Это внутреннее сопротивление источника ЭДС. Оно достаточно мало, в большинстве случаев при практических расчетах им можно пренебречь (при условии, что R>>r – сопротивление цепи много больше внутреннего сопротивления источника). Однако, когда они соизмеримы, пренебрегать величиной r нельзя.

Как вариант можно рассмотреть случай, при котором R=0 (короткое замыкание). Тогда приведенная формула закона Ома для полной цепи примет вид: I=E/r, то есть величина внутреннего сопротивления будет определять ток короткого замыкания. Такая ситуация вполне может быть реальной. Закон Ома рассмотрен здесь достаточно бегло, но приведенных формул достаточно для проведения большинства расчетов, примеры которых, по мере размещения других материалов я буду приводить.

Полноценную цепь составляет уже участок (участки), а также источник ЭДС. То есть, фактически к существующему резистивному компоненту участка цепи добавляется внутреннее сопротивление источника ЭДС. Поэтому логичным является некоторое изменение выше рассмотренной формулы:

I = U / (R + r)

Конечно, значение внутреннего сопротивления ЭДС в законе Ома для полной электрической цепи можно считать ничтожно малым, правда во многом это значение сопротивления зависит от структуры источника ЭДС. Тем не менее, при расчетах сложных электронных схем, электрических цепей с множеством проводников, наличие дополнительного сопротивления является важным фактором.

Как для участка цепи, так и для полной схемы следует учитывать естественный момент – использование тока постоянной или переменной величины. Если отмеченные выше моменты, характерные для закона Ома, рассматривались с точки зрения использования постоянного тока, соответственно с переменным током всё выглядит несколько иначе.

Для переменного тока

Переменный ток отличается от постоянного тем, что он изменяется с определенными временными периодами. Конкретно он изменяет свое значение и направление. Чтобы применить закон Ома здесь нужно учитывать, что сопротивление в цепи с постоянным током может отличатся от сопротивления в цепи с током переменным. И отличается оно в том случае если в цепи применены компоненты с реактивным сопротивлением. Реактивное сопротивление может быть индуктивным (катушки, трансформаторы, дроссели) и емкостными (конденсатор).

Если мы схематически представим, как с течением времени меняются эти два значения, у нас получится синусоида. И напряжение, и сила тока от нуля поднимаются до максимального значения, затем, опускаясь, проходят через нулевое значение и достигают максимального отрицательного значения. После этого снова поднимаются через нуль до максимального значения и так далее. Когда говорится, что сила тока или напряжение имеет отрицательное значение, здесь имеется ввиду, что они движутся в обратном направлении.


Весь процесс происходит с определенной периодичностью. Та точка, где значение напряжения или силы тока из минимального значения поднимаясь к максимальному значению проходит через нуль называется фазой.

Для замкнутой цепи

На самом деле, это только предисловие. Вернемся к реактивному и активному сопротивлению. Отличие активного сопротивления от реактивного в том, что в цепи с активным сопротивлением фаза тока совпадает с фазой напряжения. То есть, и значение силы тока, и значение напряжения достигают максимума в одном направлении одновременно. В таком случае наша формула для расчета напряжения, сопротивления или силы тока не меняется.


Следствия закона Ома.

Если же цепь содержит реактивное сопротивление, фазы тока и напряжения сдвигаются друг от друга на ¼ периода. Это означает, что, когда сила тока достигнет максимального значения, напряжение будет равняться нулю и наоборот. Когда применяется индуктивное сопротивление, фаза напряжения «обгоняет» фазу тока. Когда применяется емкостное сопротивление, фаза тока «обгоняет» фазу напряжения.

Формула для расчета падения напряжения на индуктивном сопротивлении:

U = I ⋅ ωL

Где L – индуктивность реактивного сопротивления, а ω – угловая частота (производная по времени от фазы колебания).

Формула для расчета падения напряжения на емкостном сопротивлении:

U = I / ω ⋅ С

С – емкость реактивного сопротивления.

Эти две формулы – частные случаи закона Ома для переменных цепей.

Полный же будет выглядеть следующем образом:

I = U / Z

Здесь Z – полное сопротивление переменной цепи известное как импеданс.

Сфера применения

Закон Ома не является базовым законом в физике, это лишь удобная зависимость одних значений от других, которая подходит почти в любых ситуациях на практике. Поэтому проще будет перечислить ситуации, когда закон может не срабатывать:

  • Если есть инерция носителей заряда, например, в некоторых высокочастотных электрических полях;
  • В сверхпроводниках;
  • Если провод нагревается до такой степени, что вольтамперная характеристика перестает быть линейной. Например, в лампах накаливания;
  • В вакуумных и газовых радиолампах;
  • В диодах и транзисторах.



Закон Ома для замкнутой цепи

Подобная интерпретация подразумевает наличие источника питания, а также проводника, по которому протекает ток. В этом случае, помимо сопротивления на отдельно взятом участке следует учитывать и то, которое возникает в ИП. Учитывая эти факторы, можно сказать, что сила тока будет равна отношению электродвижущей силы к сумме сопротивлений.

I = E/ Rвн+r,

где Е – ЭДС, Rвн – внешнее сопротивление, а r соответственно внутреннее.

Закон Ома для замкнутой цепи можно объяснить доступным языком. Электродвижущая сила по определению должна полноценно обеспечивать постоянную разницу потенциалов, и эта сила может иметь неприродное происхождение: химическое, если в качестве источника используется батарейка или механическая, в случае подключения к электрической цепи генератора. При подключении медной проволоки с идентичным сечением к батарейке и аккумулятору. Эффект должен быть таким, что по этому проводнику, в котором сопротивление практически отсутствует, должен пойти ток с величиной, стремящейся к бесконечности. Однако этого не происходит и разница в показателях будет существенной, а во втором случае, проволока и вовсе может перегореть. Именно поэтому в расчет берется внутреннее сопротивление источника питания, чтобы описать подобное явление.

Закон Ома для неоднородного участка цепи

Перед тем, как записать формулу для подобной интерпретации закона, следует разобраться в таких понятиях, как линейные и нелинейные участки цепи.

Если сопротивление никаким образом не зависит от тока и подаваемого напряжения, то с ростом второго параметра, первый будет прямо пропорционально возрастать и наоборот, то есть зависимость можно описать прямой линией. Подобная зависимость относится к линейным участкам цепи и сопротивление имеет аналогичное название.

Однако вышеизложенный вариант считается идеальным и его можно смоделировать лишь в идеальных условиях, что фактически невозможно, ведь, как минимум, окружающая среда вносит свои коррективы. В этом случае, рост напряжения не будет прямо пропорциональным силе тока и на графике зависимость будет изображаться в виде кривой.

На рисунке изображено два графика, первый из которых описывает линейную зависимость, а второй нелинейную.

Чтобы отчетливо понимать разницу между этими понятиями, рассмотрим принцип работы обычной электрической лампы накаливания. При прохождении тока по нити, температура в значительной степени повышается, что приводит к заметному росту сопротивления. Соответственно, при возрастании напряжения, сила тока будет увеличиваться медленнее, то есть не линейно.

Примечание: в некоторых ситуациях, некоторыми внешними факторами пренебрегают по причине того, что они очень незначительны и в числовом эквиваленте никоим образом не могут повлиять на общую картину. Это значит, что нелинейная зависимость на графике фактически совпадает с линейной.

Учитывая вышесказанное, можно установить следующую зависимость:

I = U/ R = (f1 – f2) + E/ R,

Где f1 и f2 – потенциалы (соответственно f1 – f2 называется разницей потенциалов), E – ЭДС неоднородного участка цепи, а R – суммарное сопротивление на этом же участке.

Нужно упомянуть и о том, что электродвижущая сила не всегда в этом случае будет иметь положительное значение. Если направление тока источника будет аналогичным с направлением в электрической сети, протонов будет больше, чем электронов (положительных и отрицательных частиц), то в этом случае величина E будет иметь значение со знаком «+», в иной ситуации, этот параметр будет со знаком «-».

Закон Ома для переменного тока

Если в электроцепи имеется емкость или инертность, то этот факт следует однозначно учитывать при расчётах силы тока. Они имеют собственные показатели сопротивления, что приводит к ситуации, которая будет иметь переменный характер. В случае Закона Ома для переменного тока формула записывается следующим образом:

I = U/ Z, где

I – сила тока, U – напряжение, а Z – суммарное значение сопротивления на всех участках электрической цепи (этот параметр именуется еще, как импеданс).

Как говорилось изначально, закон Ома считается эмпирическим. Это обозначает то, что он может не всегда работать и выполнять вычисления на его основе не представляется возможным. Подобная ситуация может сложиться в нескольких случаях:

  • в ситуации, когда электросеть имеет высокую частоту и электромагнитное поле может сильно изменяться за короткие промежутки времени;
  • при наличии проводников, которые обладают свойствами сверхпроводимости, расположенных в условиях низких температурных показателей;
  • при перегреве проводника под воздействием проходящего по нему тока, отношение напряжения и сопротивления может носить переменный, неоднородный характер;
  • если проводник (диэлектрик) находится под высоким напряжением;
  • светодиодных лампах;
  • в полупроводниках и аналогичных устройствах.

На основе этого закона, можно произвести вывод некоторых формул математическим путем. С их помощью можно производить разнообразные расчеты.

Метода треугольника закона Ома

Закон Ома – очень простой и полезный инструмент для анализа электрических цепей. Он так часто используется при изучении электричества и электроники, что студент должен запомнить его. Если вы не очень хорошо умеете работать с формулами, то для его запоминания существует простой прием, помогающий использовать его для любой величины, зная две других. Сначала расположите буквы E, I и R в виде треугольника следующим образом:


Рисунок 5 – Треугольник закона Ома

Если вы знаете E и I и хотите определить R, просто удалите R с картинки и посмотрите, что осталось:


Рисунок 6 – Закон Ома для определения R

Если вы знаете E и R и хотите определить I, удалите I и посмотрите, что осталось:


Рисунок 7 – Закон Ома для определения I

Наконец, если вы знаете I и R и хотите определить E, удалите E и посмотрите, что осталось:


Рисунок 8 – Закон Ома для определения E

В конце концов, вам придется научиться работать с формуми, чтобы серьезно изучать электричество и электронику, но этот совет может облегчить запоминание ваших первых вычислений. Если вам удобно работать с формулами, всё, что вам нужно сделать, это зафиксировать в памяти E = IR и вывести из нее две другие формулы, когда они вам понадобятся!

Значение Закона Ома

Закон Ома определяет силу тока в электрической цепи при заданном напряжении и известном сопротивлении.  Он позволяет рассчитать тепловые, химические и магнитные действия тока, так как они зависят от силы тока.

Закон Ома является чрезвычайно полезным в технике(электронной/электрической), поскольку он касается трех основных электрических величин: тока, напряжения и сопротивления. Он показывает, как эти три величины являются взаимозависимыми на макроскопическом уровне.

Если бы было можно охарактеризовать закон Ома простыми словами, то наглядно это выглядело бы так:

Из закона Ома вытекает, что замыкать обычную осветительную сеть проводником малого сопротивления опасно. Сила тока окажется настолько большой, что это может иметь тяжелые последствия.

Задача 1.1

Рассчитать силу тока, проходящую по медному проводу длиной 100 м, площадью поперечного сечения 0,5 мм2, если к концам провода приложено напряжение 12 B.

Задачка простая, заключается в нахождении сопротивления медной проволоки с последующим расчетом силы тока по формуле закона Ома для участка цепи. Приступим.

Сопротивление

Представьте, что есть труба, в которую затолкали камни. Вода, которая протекает по этой трубе, станет течь медленнее, потому что у нее появилось сопротивление. Точно также будет происходить с электрическим током.

  • Сопротивление — физическая величина, которая показывает способность проводника пропускать электрический ток. Чем выше сопротивление, тем ниже эта способность.

Теперь сделаем «каменный участок» длиннее, то есть добавим еще камней. Воде будет еще сложнее течь.

Сделаем трубу шире, оставив количество камней тем же — воде полегчает, поток увеличится.

Теперь заменим шероховатые камни, которые мы набрали на стройке, на гладкие камушки из моря. Через них проходить тоже легче, а значит сопротивление уменьшается.

Электрический ток реагирует на эти параметры аналогичным образом: при удлинении проводника сопротивление увеличивается, при увеличении поперечного сечения (ширины) проводника сопротивление уменьшается, а если заменить материал — изменится в зависимости от материала. 2.

Знайте!СИ — международная система единиц. «Перевести в СИ» означает перевод всех величин в метры, килограммы, секунды и другие единицы измерения без приставок. Исключение составляет килограмм с приставкой «кило».

  • Удельное сопротивление проводника — это физическая величина, которая показывает способность материала пропускать электрический ток. Это табличная величина, она зависит только от материала.

Таблица удельных сопротивлений различных материалов

Удельное сопротивление

ρ, Ом*мм2/м

Удельное сопротивление

ρ, Ом*мм2/м

Алюминий0,028
Бронза0,095 – 0,1
Висмут1,2
Вольфрам0,05
Железо0,1
Золото0,023
Иридий0,0474
Константан ( сплав Ni-Cu + Mn)0,5
Латунь0,025 – 0,108
Магний0,045
Манганин (сплав меди марганца и никеля – приборный)0,43 – 0,51
Медь0,0175
Молибден0,059
Нейзильбер (сплав меди цинка и никеля)0,2
Натрий0,047
Никелин ( сплав меди и никеля)0,42
Никель0,087
Нихром ( сплав никеля хрома железы и марганца)1,05 – 1,4
Олово0,12
Платина0.
107
Ртуть0,94
Свинец0,22
Серебро0,015
Сталь0,103 – 0,137
Титан0,6
Хромаль1,3 – 1,5
Цинк0,054
Чугун0,5-1,0

Резистор

Все реальные проводники имеют сопротивление, но его стараются сделать незначительным. В задачах вообще используют словосочетание «идеальный проводник», а значит лишают его сопротивления.

Из-за того, что проводник у нас «кругом-бегом-такой-идеальный», чаще всего за сопротивление в цепи отвечает резистор. Это устройство, которое нагружает цепь сопротивлением.

Вот так резистор изображается на схемах:

В школьном курсе физики используют Европейское обозначение, поэтому запоминаем только его. Американское обозначение можно встретить, например, в программе Micro-Cap, в которой инженеры моделируют схемы.

Вот так резистор выглядит в естественной среде обитания:

Полосочки на нем показывают его сопротивление.

На сайте компании Ekits, которая занимается продажей электронных модулей, можно выбрать цвет резистора и узнать значение его сопротивления:

Источник: сайт компании Ekits

О том, зачем дополнительно нагружать сопротивлением цепь, мы поговорим в этой же статье чуть позже.

Реостат

Есть такие выключатели, которые крутишь, а они делают свет ярче-тусклее. В такой выключатель спрятан резистор с переменным сопротивлением — реостат.

Стрелка сверху — это ползунок. По сути, он отсекает ту часть резистора, которая находится от него справа. То есть, если мы двигаем ползунок вправо — мы увеличиваем длину резистора, а значит и сопротивление. И наоборот — двигаем влево и уменьшаем.

По формуле сопротивления это очень хорошо видно, так как длина проводника находится в числителе:

Сопротивление

R = ρ l/S

R — сопротивление [Ом]

l — длина проводника [м]

S — площадь поперечного сечения [мм^2]

ρ — удельное сопротивление [Ом*мм^2/м]

Закон Ома для участка цепи

С камушками в трубе все понятно, но не только же от них зависит сила, с которой поток воды идет по трубе — от насоса, которым мы эту воду качаем, тоже зависит. Чем сильнее качаем, тем больше течение. В электрической цепи функцию насоса выполняет источник тока.

Например, источником может быть гальванический элемент (привычная батарейка). Батарейка работает на основе химических реакций внутри нее. Эти реакции выделяют энергию, которая потом передается электрической цепи.

У любого источника обязательно есть полюса — «плюс» и «минус». Полюса — это его крайние положения, по сути клеммы, к которым присоединяется электрическая цепь. Собственно, ток как раз течет от «+» к «-».

У нас уже есть две величины, от которых зависит электрический ток в цепи — напряжение и сопротивление. Кажется, пора объединять их в закон.

Сила тока в участке цепи прямо пропорциональна напряжению на его концах и обратно пропорциональна его сопротивлению.

Математически его можно описать вот так:

Закон Ома для участка цепи

I = U/R

I — сила тока [A]

U — напряжение [В]

R — сопротивление [Ом]

Напряжение измеряется в Вольтах и показывает разницу между двумя точками цепи: от этой разницы зависит, насколько сильно будет течь ток — чем больше разница, тем выше напряжение и ток будет течь сильнее. 2/м

Обратимся к таблице удельных сопротивлений материалов, чтобы выяснить, из какого материала сделана эта нить накаливания.

Закон Ома для полной цепи

Мы разобрались с законом Ома для участка цепи. А теперь давайте узнаем, что происходит, если цепь полная: у нее есть источник, проводники, резисторы и другие элементы.

В таком случае вводится Закон Ома для полной цепи: сила тока в полной цепи равна отношению ЭДС цепи к ее полному сопротивлению.

Так, стоп. Слишком много незнакомых слов — разбираемся по-порядку.

Что такое ЭДС и откуда она берется

ЭДС расшифровывается, как электродвижущая сила. Обозначается греческой буквой ε и измеряется, как и напряжение, в Вольтах.

  • ЭДС — это сила, которая движет заряженные частицы в цепи. Она берется из источника тока. Например, из батарейки.

Химическая реакция внутри гальванического элемента (это синоним батарейки) происходит с выделением энергии в электрическую цепь. Именно эта энергия заставляет частицы двигаться по проводнику.

Зачастую напряжение и ЭДС приравнивают и говорят, что это одно и то же. Формально, это не так, но при решении задач чаще всего и правда нет разницы, так как эти величины обе измеряются в Вольтах и определяют очень похожие по сути своей процессы.

В виде формулы Закон Ома для полной цепи будет выглядеть следующим образом:

Закон Ома для полной цепи

I = ε/(R + r)

I — сила тока [A]

ε — ЭДС [В]

R — сопротивление [Ом]

r — внутреннее сопротивление источника [Ом]

Любой источник не идеален. В задачах это возможно («источник считать идеальным», вот эти вот фразочки), но в реальной жизни — точно нет. В связи с этим у источника есть внутреннее сопротивление, которое мешает протеканию тока.

Решим задачу на полную цепь.

Задачка

Найти силу тока в полной цепи, состоящей из одного резистора сопротивлением 3 Ом и источником с ЭДС равной 4 В и внутренним сопротивлением 1 Ом

Решение:

Возьмем закон Ома для полной цепи:

I = ε/(R + r)

Подставим значения:

I = 4/(3+1) = 1 A

Ответ: сила тока в цепи равна 1 А.

Когда «сопротивление бесполезно»

Электрический ток — умный и хитрый парень. Если у него есть возможность обойти резистор и пойти по идеальному проводнику без сопротивления, он это сделает. При этом с резисторами просто разных номиналов это не сработает: он не пойдет просто через меньшее сопротивление, а распределится согласно закону Ома — больше тока пойдет туда, где сопротивление меньше, и наоборот.

А вот на рисунке ниже сопротивление цепи равно нулю, потому что ток через резистор не пойдет.

Ток идет по пути наименьшего сопротивления.

Теперь давайте посмотрим на закон Ома для участка цепи еще раз.

Закон Ома для участка цепи

I = U/R

I — сила тока [A]

U — напряжение [В]

R — сопротивление [Ом]

Подставим сопротивление, равное 0. Получается, что знаменатель равен нулю, а на математике говорят, что на ноль делить нельзя. Но мы вам раскроем страшную тайну, только не говорите математикам: на ноль делить можно. Если совсем упрощать такое сложное вычисление (а именно потому что оно сложное, мы всегда говорим, что его нельзя производить), то получится бесконечность.

То есть:

I = U/0 = ∞

Такой случай называют коротким замыканием — когда величина силы тока настолько велика, что можно устремить ее к бесконечности. В таких ситуациях мы видим искру, бурю, безумие — и все ломается.

Это происходит, потому что две точки цепи имеют между собой напряжение (то есть между ними есть разница). Это как если вдоль реки неожиданно появляется водопад. Из-за этой разницы возникает искра, которую можно избежать, поставив в цепь резистор.

Именно во избежание коротких замыканий нужно дополнительное сопротивление в цепи.

Формула Закона Джоуля-Ленца

Величину резистора для изготовления блока нагрузки для блока питания компьютера мы рассчитали, но нужно еще определить какой резистор должен быть мощности? Тут поможет другой закон физики, который, независимо друг от друга открыли одновременно два ученых физика. В 1841 году Джеймс Джоуль, а в 1842 году Эмиль Ленц. Этот закон и назвали в их честь – Закон Джоуля-Ленца.

Потребляемая нагрузкой мощность прямо пропорциональна приложенной величине напряжения и протекающей силе тока. Другими словами, при изменении величины напряжения и тока будет пропорционально будет изменяться и потребляемая мощность.


где P – мощность, измеряется в ваттах и обозначается Вт;U – напряжение, измеряется в вольтах и обозначается буквой В;I – сила ток, измеряется в амперах и обозначается буквой А.

Зная напряжения питания и силу тока, потребляемую электроприбором, можно по формуле определить, какую он потребляет мощность. Достаточно ввести данные в окошки ниже приведенного онлайн калькулятора.

Закон Джоуля-Ленца позволяет также узнать силу тока, потребляемую электроприбором зная его мощность и напряжение питания. Величина потребляемого тока необходима, например, для выбора сечения провода при прокладке электропроводки или для расчета номинала.

Например, рассчитаем потребляемый ток стиральной машины. По паспорту потребляемая мощность составляет 2200 Вт, напряжение в бытовой электросети составляет 220 В. Подставляем данные в окошки калькулятора, получаем, что стиральная машина потребляет ток величиной 10 А.

Еще один пример, Вы решили в автомобиле установить дополнительную фару или усилитель звука. Зная потребляемую мощность устанавливаемого электроприбора легко рассчитать потребляемый ток и правильно подобрать сечение провода для подключения к электропроводке автомобиля. Допустим, дополнительная фара потребляет мощность 100 Вт (мощность установленной в фару лампочки), бортовое напряжение сети автомобиля 12 В. Подставляем значения мощности и напряжения в окошки калькулятора, получаем, что величина потребляемого тока составит 8,33 А.

Разобравшись всего в двух простейших формулах, Вы легко сможете рассчитать текущие по проводам токи, потребляемую мощность любых электроприборов – практически начнете разбираться в основах электротехники.

Преобразованные формулы Закона Ома и Джоуля-Ленца

Встретил в Интернете картинку в виде круглой таблички, в которой удачно размещены формулы Закона Ома и Джоуля-Ленца и варианты математического преобразования формул. Табличка представляет собой не связанные между собой четыре сектора и очень удобна для практического применения

По таблице легко выбрать формулу для расчета требуемого параметра электрической цепи по двум другим известным. Например, нужно определить ток потребления изделием по известной мощности и напряжению питающей сети. По таблице в секторе тока видим, что для расчета подойдет формула I=P/U.

А если понадобится определить напряжение питающей сети U по величине потребляемой мощности P и величине тока I, то можно воспользоваться формулой левого нижнего сектора, подойдет формула U=P/I.

Подставляемые в формулы величины должны быть выражены в амперах, вольтах, ваттах или Омах.

Применение закона Ома на практике

На практике часто приходится определять не силу тока I, а величину сопротивления R. Преобразовав формулу Закона Ома, можно рассчитать величину сопротивления R, зная протекающий ток I и величину напряжения U.

Величину сопротивления может понадобится рассчитать, например, при изготовлении блока нагрузок для проверки блока питания компьютера. На корпусе блока питания компьютера обычно есть табличка, в которой приведен максимальный ток нагрузки по каждому напряжению. Достаточно в поля калькулятора ввести данные величины напряжения и максимальный ток нагрузки и в результате вычисления получим величину сопротивления нагрузки для данного напряжения. Например, для напряжения +5 В при максимальной величине тока 20 А, сопротивление нагрузки составит 0,25 Ом.

Таблица-шпаргалка

Используя закон Ома для участка цепи, а также формулу для мощности электрического тока: P = U*I – я подготовил для вас полезную таблицу-шпаргалку, которая позволяет соотносить между собой сопротивление (R), силу тока (I), напряжение (U) и мощность электрического тока (P). Будет точно полезно не только школьникам!

Известные величиныR (сопротивление)I (сила тока)U (напряжение)P (мощность)
Ток и сопротивлениеU = I × RP = I2 × R
Напряжение и токR = U / IP = U × I
Мощность и токR = P / I2U = P / I
Напряжение и сопротивлениеI = U / RP = U2 / R
Мощность и сопротивлениеI = P / R
Напряжение и мощностьR = U2 / RI = P / U

Последовательное и параллельное включение элементов

Для элементов электрической цепи (участка цепи) характерным моментом является последовательное либо параллельное соединение. Соответственно, каждый вид соединения сопровождается разным характером течения тока и подводкой напряжения. На этот счёт закон Ома также применяется по-разному, в зависимости от варианта включения элементов.

Цепь последовательно включенных резистивных элементов

Применительно к последовательному соединению (участку цепи с двумя компонентами) используется формулировка:

  • I = I1= I2 ;
  • U = U1+ U2 ;
  • R = R1+ R2

Такая формулировка явно демонстрирует, что, независимо от числа последовательно соединенных резистивных компонентов, ток, текущий на участке цепи, не меняет значения. Величина напряжения, приложенного к действующим резистивным компонентам схемы, является суммой и составляет в целом значение источника ЭДС.

При этом напряжение на каждом отдельном компоненте равно: Ux = I * Rx. Общее сопротивление следует рассматривать как сумму номиналов всех резистивных компонентов цепи.

Цепь параллельно включенных резистивных элементов

На случай, когда имеет место параллельное включение резистивных компонентов, справедливой относительно закона немецкого физика Ома считается формулировка:

  • I = I1+ I2 … ;
  • U = U1= U2 … ;
  • 1 / R = 1 / R1+ 1 / R2 + …

Не исключаются варианты составления схемных участков «смешанного» вида, когда используется параллельное и последовательное соединение. Для таких вариантов расчет обычно ведется изначальным расчетом резистивного номинала параллельного соединения. Затем к полученному результату добавляется номинал резистора, включенного последовательно.

Интегральная и дифференциальная формы закона

Все вышеизложенные моменты с расчетами применимы к условиям, когда в составе электрических схем используются проводники, так сказать, «однородной» структуры. Между тем на практике нередко приходится сталкиваться с построением схематики, где на различных участках структура проводников меняется. К примеру, используются провода большего сечения или, напротив, меньшего, сделанные на основе разных материалов.


Для учёта таких различий существует вариация, так называемого, «дифференциально-интегрального закона Ома». Для бесконечно малого проводника рассчитывается уровень плотности тока в зависимости от напряженности и величины удельной проводимости.

Под дифференциальный расчет берется формула: J = ό * E. Для интегрального расчета, соответственно, формулировка: I * R = φ1 – φ2 + έ Однако эти примеры скорее уже ближе к школе высшей математики и в реальной практике простого электрика фактически не применяются.

Параллельное и последовательное соединение

В электрике элементы соединяются либо последовательно — один за другим, либо параллельно — это когда к одной точке подключены несколько входов, к другой — выходы от тех же элементов.

Закон Ома для параллельного и последовательного соединения

Последовательное соединение

Как работает закон Ома для этих случаев? При последовательном соединении сила тока, протекающая через цепочку элементов, будет одинаковой. Напряжение участка цепи с последовательно подключенными элементами считается как сумма напряжений на каждом участке. Как можно это объяснить? Протекание тока через элемент — это перенос части заряда с одной его части в другую. То есть, это определенная работа. Величина этой работы и есть напряжение. Это физический смысл напряжения. Если с этим понятно, двигаемся дальше.

Последовательное соединение и параметры этого участка цепи

При последовательном соединении приходится переносить заряд по очереди через каждый элемент. И на каждом элементе это определенный «объем» работы. А чтобы найти объем работы на всем участке цепи, надо работу на каждом элементе сложить. Вот и получается, что общее напряжение — это сумма напряжений на каждом из элементов.

Точно так же — при помощи сложения — находится и общее сопротивление участка цепи. Как можно это себе представить? Ток, протекая по цепочке элементов, последовательно преодолевает все сопротивления. Одно за другим. То есть чтобы найти сопротивление, которое он преодолел, надо сопротивления сложить. Примерно так. Математический вывод более сложен, а так понять механизм действия этого закона проще.

Параллельное соединение

Параллельное соединение — это когда начала проводников/элементов сходятся в одной точке, а в другой — соединены их концы. Постараемся объяснить законы, которые справедливы для соединений этого типа. Начнем с тока. Ток какой-то величины подается в точку соединения элементов. Он разделяется, протекая по всем проводникам. Отсюда делаем вывод, что общий ток на участке равен сумме тока на каждом из элементов: I = I1 + I2 + I3.

Теперь относительно напряжения. Если напряжение — это работа по перемещению заряда, тоо работа, которая необходима на перемещение одного заряда будет одинакова на любом элементе. То есть, напряжение на каждом параллельно подключенном элементе будет одинаковым. U = U1=U2=U3. Не так весело и наглядно, как в случае с объяснением закона Ома для участка цепи, но понять можно.

Законы для параллельного соединения

Для сопротивления все несколько сложнее. Давайте введем понятие проводимости. Это характеристика, которая показывает насколько легко или сложно заряду проходить по этому проводнику. Понятно, что чем меньше сопротивление, тем проще току будет проходить. Поэтому проводимость — G — вычисляется как величина обратная сопротивлению. В формуле это выглядит так: G = 1/R.

Для чего мы говорили о проводимости? Потому что общая проводимость участка с параллельным соединением элементов равна сумме проводимости для каждого из участков. G = G1 + G2 + G3 — понять несложно. Насколько легко току будет преодолеть этот узел из параллельных элементов, зависит от проводимости каждого из элементов. Вот и получается, что их надо складывать.

Теперь можем перейти к сопротивлению. Так как проводимость — обратная к сопротивлению величина, можем получить следующую формулу: 1/R = 1/R1 + 1/R2 + 1/R3.

Что нам дает параллельное и последовательное соединение?

Теоретические знания — это хорошо, но как их применить на практике? Параллельно и последовательно могут соединяться элементы любого типа. Но мы рассматривали только простейшие формулы, описывающие линейные элементы. Линейные элементы — это сопротивления, которые еще называют «резисторы». Итак, вот как можно использовать полученные знания:

  • Если в наличии нет резистора большого номинала, но есть несколько более «мелких», нужное сопротивление можно получить соединив последовательно несколько резисторов. Как видите, это полезный прием.
  • Для продления срока жизни батареек, их можно соединять параллельно. Напряжение при этом, согласно закону Ома, останется прежним (можно убедиться, измерив напряжение мультиметром). А «срок жизни» сдвоенного элемента питания будет значительно больше, нежели у двух элементов, которые сменят друг друга. Только обратите внимание: параллельно соединять можно только источники питания с одинаковым потенциалом. То есть, севшую и новую батарейки соединять нельзя. Если все-таки соединить, та батарейка которая имеет больший заряд, будет стремиться зарядить менее заряженную. В результате общий их заряд упадет до низкого значения.
    Практическое применение закона Ома: можно создавать источники питания с нужным напряжением и силой тока

В общем, это наиболее распространенные варианты использования этих соединений.

Интегральная и дифференциальная формы закона

Все вышеизложенные моменты с расчетами применимы к условиям, когда в составе электрических схем используются проводники, так сказать, «однородной» структуры. Между тем на практике нередко приходится сталкиваться с построением схематики, где на различных участках структура проводников меняется. К примеру, используются провода большего сечения или, напротив, меньшего, сделанные на основе разных материалов.
Для учёта таких различий существует вариация, так называемого, «дифференциально-интегрального закона Ома». Для бесконечно малого проводника рассчитывается уровень плотности тока в зависимости от напряженности и величины удельной проводимости.

Под дифференциальный расчет берется формула: J = ό * E. Для интегрального расчета, соответственно, формулировка: I * R = φ1 – φ2 + έ Однако эти примеры скорее уже ближе к школе высшей математики и в реальной практике простого электрика фактически не применяются.

Источники

  • https://ElectroInfo.net/teorija/vse-o-zakone-oma-prostymi-slovami-s-primerami-dlja-chajnikov.html
  • https://uelektrika.ru/osnovy-yelektrotekhniki/zakon-oma-dlya-yelektricheskoy-cepi/
  • https://serp1.ru/kak-ponjat-zakon-oma-prostoe-objasnen/
  • https://skysmart.ru/articles/physics/zakon-oma
  • https://mnogoformul.ru/zakon-oma
  • https://www.asutpp.ru/zakon-oma-dlya-uchastka-tsepi.html

Закон Ома для участка цепи и полной цепи. Общие методы эквивалентного — презентация на Slide-Share.ru 🎓

1

Первый слайд презентации: Закон Ома для участка цепи и полной цепи. Общие методы эквивалентного преобразования схем

Изображение слайда

2

Слайд 2

Закон Ома, основанный на опытах, представляет собой в электротехнике основной закон, который устанавливает связь силы электрического тока с сопротивлением и напряжением. Строгая формулировка закона Ома для участка цепи может быть записана так: сила тока в проводнике прямо пропорциональна напряжению на его концах (разности потенциалов) и обратно пропорциональна сопротивлению этого проводника. Формула закона Ома записывается в следующем виде:

Изображение слайда

3

Слайд 3

где I – сила тока в проводнике, единица измерения силы тока – ампер [А]; U – электрическое напряжение (разность потенциалов), единица измерения напряжения- вольт [В]; R – электрическое сопротивление проводника, единица измерения электрического сопротивления – ом [Ом]. Согласно закону Ома, увеличение напряжения, например, в два раза при фиксированном сопротивлении проводника, приведёт к увеличению силы тока также в два раза

Изображение слайда

4

Слайд 4

И напротив, уменьшение тока в два раза при фиксированном напряжении будет означать, что сопротивление увеличилось в два раза. Существует мнемоническое правило для запоминания этого закона, которое можно назвать треугольник Ома. Изобразим все три характеристики (напряжение, сила тока и сопротивление) в виде треугольника. В вершине которого находится напряжение, в нижней левой части – сила тока, а в правой – сопротивление.

Изображение слайда

5

Слайд 5

Правило работы такое: закрываем пальцем величину в треугольнике, которую нужно найти, тогда две оставшиеся дадут верную формулу для поиска закрытой.

Изображение слайда

6

Слайд 6

Рис.1. Участок цепи Рис.2. Полная цепь Закон Ома для полной цепи: сила тока I полной электрической цепи равна ЭДС (электродвижущей силе) источника тока Е, деленной на полное сопротивление цепи ( R + r). Полное сопротивление цепи равно сумме сопротивлений внешней цепи R и внутреннего r источника тока. Формула закона. На рис. 1 и 2 приведены схемы электрических цепей.

Изображение слайда

7

Слайд 7

Проводники в электрических цепях могут соединяться последовательно и параллельно. Смешанное соединение сочетает оба эти соединения. Сопротивление, при включении которого вместо всех других проводников, находящихся между двумя точками цепи, ток и напряжение остаются неизменными, называют эквивалентным сопротивлением этих проводников. Последовательным называется соединение, при котором каждый проводник соединяется только с одним предыдущим и одним последующим проводниками. Как следует из первого правила Кирхгофа, при последовательном соединении проводников сила электрического тока, протекающего по всем проводникам, одинакова (на основании закона

Изображение слайда

8

Слайд 8

сохранения заряда ). 1. При последовательном соединении проводников (рис. 1) сила тока во всех проводниках одинакова: I1 = I2 = I3 = I Рис. 1. Последовательное соединение двух проводников. 2. Согласно закону Ома, напряжения U 1 и U 2 на проводниках равны U 1  =  IR 1,   U 2  =  IR 2, U 3  =  IR 3. Напряжение при последовательном соединении проводников равно сумме напряжений на отдельных участках (проводниках) электрической цепи. U  =  U 1 + U 2  +  U 3

Изображение слайда

9

Слайд 9

По закону Ома, напряжения U 1, U 2 на проводниках равны U 1  =  IR 1,   U 2  =  IR 2, В соответствии вторым правилом Кирхгофа напряжение на всем участке: U  =  U 1 + U 2  = IR 1 +  IR 2 = I(R 1 + R 2 )= I·R. Получаем: R  = R 1  +  R 2 Общее напряжение U на проводниках равно сумме напряжений U 1, U 2, U 3 равно: U  = U 1  +  U 2  +  U 3  =  I· ( R 1  +  R 2 +  R 3 )  =  IR где R ЭКВ – эквивалентное сопротивление всей цепи. Отсюда: R ЭКВ = R 1  +  R 2  +  R 3 При последовательном соединении эквивалентное сопротивление цепи равно сумме сопротивлений отдельных участков цепи : R   ЭКВ = R 1  +  R 2  +  R 3 +… Этот результат справедлив для любого числа последовательно соединенных проводников.

Изображение слайда

10

Слайд 10

Из закона Ома следует: при равенстве сил тока при последовательном соединении: ,. Отсюда или, т. е. напряжения на отдельных участках цепи прямо пропорциональны сопротивлениям участков. При последовательном соединении n одинаковых проводников общее напряжение равно произведению напряжению одного U 1 на их количество n : U ПОСЛЕД = n ·U 1. Аналогично для сопротивлений : R ПОСЛЕД = n· R 1 При размыкании цепи одного из последовательно соединенных потребителей ток исчезает во всей цепи, поэтому последовательное соединение на практике не всегда удобно.

Изображение слайда

11

Слайд 11

Параллельным называется соединение проводников, при котором одни их концы образуют узел А, а другие – узел В. Узловые точки (узлы) – это точки, в которых сходятся не менее трех проводников. Параллельно соединенные проводники составляют разветвление, а каждая из них называется ветвью. 1. При параллельном соединении напряжения на всех ветвях и на всем разветвлении одинаковы, (как разности потенциалов двух точек А и В): U 1 = U 2 = U. По первому правилу Кирхгофа (алгебраическая сумма токов, сходящихся в одном узле, равно нулю), имеем: 2. Сумма токов I 1  +  I 2, протекающих по ветвям, равна силе тока I в неразветвленной части цепи: I  =  I 1  +  I 2

Изображение слайда

12

Слайд 12

Записывая на основании закона Ома: где R – электрическое сопротивление всей цепи (из двух ветвей), получим, учитывая, что напряжения одинаковы на ветвях: Для трех проводников: или: R ЭКВ = Этот результат справедлив для любого числа параллельно включенных проводников.

Изображение слайда

13

Слайд 13

Соединение элементов электрической цепи по схемам «звезда» и «треугольник» В электротехнических и электронных устройствах элементы цепи соединяются по мостовой схеме (рис. 1.12). Сопротивления R 12, R 13, R 24, R 34 включены в плечи моста, в диагональ 1–4 включен источник питания с ЭДС Е, другая диагональ 3–4 называется измерительной диагональю моста. Рис. 1.12 Рис. 1.13

Изображение слайда

14

Слайд 14

В мостовой схеме сопротивления R 13, R 12, R 23 и R 24, R 34, R 23 соединены по схеме «треугольник». Эквивалентное сопротивление этой схемы можно определить только после замены одного из треугольников, например треугольника R 24 R 34 R 23 звездой R 2 R 3 R 4 (рис.  1.13). Такая замена будет эквивалентной, если она не вызовет изменения токов всех остальных элементов цепи. Для этого величины сопротивлений звезды должны рассчитываться по следующим соотношениям: (1.8)

Изображение слайда

15

Слайд 15

Для замены схемы «звезда» эквивалентным треугольником необходимо рассчитать сопротивления треугольника: (1.9) После проведенных преобразований (рис. 1.13) можно определить величину эквивалентного сопротивления мостовой схемы (рис. 1.12)

Изображение слайда

16

Слайд 16

Изображение слайда

17

Слайд 17

Изображение слайда

18

Последний слайд презентации: Закон Ома для участка цепи и полной цепи.

Общие методы эквивалентного

Изображение слайда

Закон ома краткое определение – Вместе мастерим

Закон Ома, основанный на опытах, представляет собой в электротехнике основной закон, который устанавливает связь силы электрического тока с сопротивлением и напряжением.

Появление смартфонов, гаджетов, бытовых приборов и прочей электротехники коренным образом изменило облик современного человека. Приложены огромные усилия, направленные на исследование физических закономерностей для улучшения старой и создания новой техники. Одной из таких зависимостей является закон Ома.

Закон Ома – полученный экспериментальным путём (эмпирический) закон, который устанавливает связь силы тока в проводнике с напряжением на концах проводника и его сопротивлением, был открыт в 1826 году немецким физиком-экспериментатором Георгом Омом.

Строгая формулировка закона Ома может быть записана так: сила тока в проводнике прямо пропорциональна напряжению на его концах (разности потенциалов) и обратно пропорциональна сопротивлению этого проводника.

Формула закона Ома записывается в следующем виде:

U – электрическое напряжение (разность потенциалов), единица измерения напряжения- вольт [В];

Согласно закону Ома, увеличение напряжения, например, в два раза при фиксированном сопротивлении проводника, приведёт к увеличению силы тока также в два раза

И напротив, уменьшение тока в два раза при фиксированном напряжении будет означать, что сопротивление увеличилось в два раза.

Рассмотрим простейший случай применения закона Ома. Пусть дан некоторый проводник сопротивлением 3 Ом под напряжением 12 В. Тогда, по определению закона Ома, по данному проводнику течет ток равный:

Существует мнемоническое правило для запоминания этого закона, которое можно назвать треугольник Ома. Изобразим все три характеристики (напряжение, сила тока и сопротивление) в виде треугольника. В вершине которого находится напряжение, в нижней левой части – сила тока, а в правой – сопротивление.

Правило работы такое: закрываем пальцем величину в треугольнике, которую нужно найти, тогда две оставшиеся дадут верную формулу для поиска закрытой.

Где и когда можно применять закон Ома?

Закон Ома в упомянутой форме справедлив в достаточно широких пределах для металлов. Он выполняется до тех пор, пока металл не начнет плавиться. Менее широкий диапазон применения у растворов (расплавов) электролитов и в сильно ионизированных газах (плазме).

Работая с электрическими схемами, иногда требуется определять падение напряжения на определенном элементе. Если это будет резистор с известной величиной сопротивления (она проставляется на корпусе), а также известен проходящий через него ток, узнать напряжение можно с помощью формулы Ома, не подключая вольтметр.

Значение Закона Ома

Закон Ома определяет силу тока в электрической цепи при заданном напряжении и известном сопротивлении.

Он позволяет рассчитать тепловые, химические и магнитные действия тока, так как они зависят от силы тока.

Закон Ома является чрезвычайно полезным в технике(электронной/электрической), поскольку он касается трех основных электрических величин: тока, напряжения и сопротивления. Он показывает, как эти три величины являются взаимозависимыми на макроскопическом уровне.

Если бы было можно охарактеризовать закон Ома простыми словами, то наглядно это выглядело бы так:

Из закона Ома вытекает, что замыкать обычную осветительную сеть проводником малого сопротивления опасно. Сила тока окажется настолько большой, что это может иметь тяжелые последствия.

Немецкий физик Георг Симон Ом (1787—1854) открыл основной закон электрической цепи.

Закон Ома для участка цепи:

Определение: Cила тока I на участке электрической цепи прямо пропорциональна напряжению U на концах участка и обратно пропорциональна его сопротивлению R.

  1. I — сила тока (в системе СИ измеряется — Ампер)
    • Сила тока в проводнике прямо пропорциональна напряжению и обратно пропорциональна сопротивлению.
    • Формула: I=frac
    • U — напряжение (в системе СИ измеряется — Вольт)

    Падение напряжения на участке проводника равно произведению силы тока в проводнике на сопротивление этого участка.

    Формула: U=IR

  2. R— электрическое сопротивление (в системе СИ измеряется — Ом).
    • Электрическое сопротивление R это отношение напряжения на концах проводника к силе тока, текущего по проводнику.
    • Формула R=frac
    • Определение единицы сопротивления — Ом

      1 Ом представляет собой электрическое сопротивление участка проводника, по которому при напряжении 1(Вольт) протекает ток 1 (Ампер).

      Закон Ома для полной цепи

      Определение: Сила тока в цепи пропорциональна действующей в цепи ЭДС и обратно пропорциональна сумме сопротивлений цепи и внутреннего сопротивления источника

      Формула I=frac <varepsilon>

      • varepsilon — ЭДС источника напряжения, В;
      • I — сила тока в цепи, А;
      • R — сопротивление всех внешних элементов цепи, Ом;
      • r — внутреннее сопротивление источника напряжения, Ом.

      Как запомнить формулы закона Ома

      Треугольник Ома поможет запомнить закон. Нужно закрыть искомую величину, и два других символа дадут формулу для её вычисления.

      .

      • U — электрическое напряжение;
      • I — сила тока;
      • P — электрическая мощность;
      • R — электрическое сопротивление

      Смотри также:

      Для закрепления своих знаний решай задания и варианты ЕГЭ по физике с ответами и пояснениями.

      Говорят: «не знаешь закон Ома – сиди дома». Так давайте же узнаем (вспомним), что это за закон, и смело пойдем гулять.

      Основные понятия закона Ома

      Как понять закон Ома? Нужно просто разобраться в том, что есть что в его определении. И начать следует с определения силы тока, напряжения и сопротивления.

      Сила тока I

      Пусть в каком-то проводнике течет ток. То есть, происходит направленное движение заряженных частиц – допустим, это электроны. Каждый электрон обладает элементарным электрическим зарядом (e= -1,60217662 × 10 -19 Кулона). В таком случае через некоторую поверхность за определенный промежуток времени пройдет конкретный электрический заряд, равный сумме всех зарядов протекших электронов.

      Отношение заряда к времени и называется силой тока. Чем больший заряд проходит через проводник за определенное время, тем больше сила тока. Сила тока измеряется в Амперах.

      Напряжение U, или разность потенциалов

      Это как раз та штука, которая заставляет электроны двигаться. Электрический потенциал характеризует способность поля совершать работу по переносу заряда из одной точки в другую. Так, между двумя точками проводника существует разность потенциалов, и электрическое поле совершает работу по переносу заряда.

      Физическая величина, равная работе эффективного электрического поля при переносе электрического заряда, и называется напряжением. Измеряется в Вольтах. Один Вольт – это напряжение, которое при перемещении заряда в 1 Кл совершает работу, равную 1 Джоуль.

      Сопротивление R

      Ток, как известно, течет в проводнике. Пусть это будет какой-нибудь провод. Двигаясь по проводу под действием поля, электроны сталкиваются с атомами провода, проводник греется, атомы в кристаллической решетке начинают колебаться, создавая электронам еще больше проблем для передвижения. Именно это явление и называется сопротивлением. Оно зависит от температуры, материала, сечения проводника и измеряется в Омах.

      Памятник Георгу Симону Ому

      Формулировка и объяснение закона Ома

      Закон немецкого учителя Георга Ома очень прост. Он гласит:

      Сила тока на участке цепи прямо пропорционально напряжению и обратно пропорциональна сопротивлению.

      Георг Ом вывел этот закон экспериментально (эмпирически) в 1826 году. Естественно, чем больше сопротивление участка цепи, тем меньше будет сила тока. Соответственно, чем больше напряжение, тем и ток будет больше.

      Кстати! Для наших читателей сейчас действует скидка 10% на любой вид работы

      Данная формулировка закона Ома – самая простая и подходит для участка цепи. Говоря “участок цепи” мы подразумеваем, что это однородный участок, на котором нет источников тока с ЭДС. Говоря проще, этот участок содержит какое-то сопротивление, но на нем нет батарейки, обеспечивающей сам ток.

      Если рассматривать закон Ома для полной цепи, формулировка его будет немного иной.

      Пусть у нас есть цепь, в ней есть источник тока, создающий напряжение, и какое-то сопротивление.

      Закон запишется в следующем виде:

      Объяснение закона Ома для полой цепи принципиально не отличается от объяснения для участка цепи. Как видим, сопротивление складывается из собственно сопротивления и внутреннего сопротивления источника тока, а вместо напряжения в формуле фигурирует электродвижущая сила источника.

      Кстати, о том, что такое что такое ЭДС, читайте в нашей отдельной статье.

      Как понять закон Ома?

      Чтобы интуитивно понять закон Ома, обратимся к аналогии представления тока в виде жидкости. Именно так думал Георг Ом, когда проводил опыты, благодаря которым был открыт закон, названный его именем.

      Представим, что ток – это не движение частиц-носителей заряда в проводнике, а движение потока воды в трубе. Сначала воду насосом поднимают на водокачку, а оттуда, под действием потенциальной энергии, она стремиться вниз и течет по трубе. Причем, чем выше насос закачает воду, тем быстрее она потечет в трубе.

      Отсюда следует вывод, что скорость потока воды (сила тока в проводе) будет тем больше, чем больше потенциальная энергия воды (разность потенциалов)

      Сила тока прямо пропорциональна напряжению.

      Теперь обратимся к сопротивлению. Гидравлическое сопротивление – это сопротивление трубы, обусловленное ее диаметром и шероховатостью стенок. Логично предположить, что чем больше диаметр, тем меньше сопротивление трубы, и тем большее количество воды (больший ток) протечет через ее сечение.

      Сила тока обратно пропорциональна сопротивлению.

      Такую аналогию можно проводить лишь для принципиального понимания закона Ома, так как его первозданный вид – на самом деле довольно грубое приближение, которое, тем не менее, находит отличное применение на практике.

      В действительности, сопротивление вещества обусловлено колебанием атомов кристаллической решетки, а ток – движением свободных носителей заряда. В металлах свободными носителями являются электроны, сорвавшиеся с атомных орбит.

      Ток в проводнике

      В данной статье мы постарались дать простое объяснение закона Ома. Знание этих на первый взгляд простых вещей может сослужить Вам неплохую службу на экзамене. Конечно, мы привели его простейшую формулировку закона Ома и не будем сейчас лезть в дебри высшей физики, разбираясь с активным и реактивным сопротивлениями и прочими тонкостями.

      Если у Вас возникнет такая необходимость, Вам с удовольствием помогут сотрудники нашего студенческого сервиса. А напоследок предлагаем Вам посмотреть интересное видео про закон Ома. Это действительно познавательно!

      Презентация на тему “закон ома для участка цепи”.


      1. Обозначение силы тока, единица измерения I, Aq, AU, В 2. Обозначение сопротивления, единица измерения R, AU, ОмR, Ом 3. Обозначение напряжения, единица измерения U, ОмI, ВU, В 4. Формула силы токаI=q/tI=qtU=A/q 5. Формула сопротивленияR= SL/pR= pL/sR= Sp/L 6. Формула напряженияU=A/qU=AqI=q/t

      Проверь себя! 1. Обозначение силы тока, единица измерения I, Aq, AU, В 2. Обозначение сопротивления, единица измерения R, AU, ОмR, Ом 3. Обозначение напряжения, единица измерения U, ОмI, ВU, В 4. Формула силы токаI=q/tI=qtU=A/q 5. Формула сопротивленияR= SL/pR= pL/sR= Sp/L 6. Формула напряженияU=A/qU=AqI=q/t 1. Обозначение силы тока, единица измерения I, Aq, AU, В 2. Обозначение сопротивления, единица измерения R, AU, ОмR, Ом 3. Обозначение напряжения, единица измерения U, ОмI, ВU, В 4. Формула силы токаI=q/tI=qtU=A/q 5. Формула сопротивленияR= SL/pR= pL/sR= Sp/L 6. Формула напряженияU=A/qU=AqI=q/t




      2. Зависимость силы тока от сопротивления Сила тока обратно пропорциональна сопротивлению I,А R,Ом 1 График – ветвь гиперболы обратная пропорциональность R ~ I 1 U, В I, А R, Ом1 Ом2 Ом4 Ом 4,5 В 4 А 2 А1 А U – const

      1. I ~ U – прямая зав – ть 2. I ~ 1/R – обратная зав – ть Георг Симон Ом (1789–1854) Закон Ома для участка цепи 1827 год Формулировка: Сила тока в участке цепи прямо пропорциональна напряжению на концах этого участка и обратно пропорциональна его сопротивлению.





      15 Рефлексия (отметьте свой вариант ответа в таблице) СужденияДаНет Не знаю На уроке я: 1)узнал много нового; 2)показал свои знания; 3)с интересом общался с учителем и одноклассниками. На уроке я чувствовал себя: 1)свободно; 2)скованно; 3)уютно. На уроке мне понравилось: 1)коллективное решение познавательных задач и вопросов; 2)наглядность; 3)другое (указать).


      Слайд 1

      Закон Ома для участка цепи
      Цель урока: установить зависимость между силой тока, напряжением на участке цепи и сопротивлением этого участка.

      Слайд 2

      Основные величины, характеризующие электрическую цепь.
      Название Что характеризует? Обозначение Единицы измерения Прибор для измер-я, Обозн-е
      Напряжение вольт [В] вольтметр
      Сила тока ампер [А] Амперметр
      Сопротивление проводник R ом [Ом] Омметр
      электрическое поле.
      U

      электрический ток в проводнике.
      I
      U
      A

      Слайд 3

      Вопрос первый: Как зависит сила тока в цепи от напряжения при постоянном сопротивлении?
      U, B I, A R, Oм
      const
      const
      const
      1. Собрать схему, представленную на рисунке. 2. Изменяя реостатом силу тока в цепи, найти соответствующее значение напряжения и заполнить таблицу. 3. Построить график зависимости силы тока от напряжения.

      Слайд 4

      Вопрос второй: Как зависит сила тока в цепи от сопротивления при постоянном напряжении?
      U, B I, A R, Oм
      const
      const
      const
      1. Собрать схему, представленную на рисунке. 2. Изменяя сопротивление участка цепи R, найти соответствующую силу тока и заполнить таблицу. 3. Построить график зависимости силы тока от сопротивления.

      Слайд 5

      Выводы:
      Первая группа – R = const, I~ U Вторая группа – U = const, I ~ 1/R.

      Слайд 6

      Тогда сможем записать

      Слайд 8

      Удобно запомнить!
      U
      I
      R

      Слайд 9

      Георг Ом (1787-1854)
      Родился 16 марта 1787 года в семье слесаря. Отец придавал большое значение образованию детей. Хотя семья постоянно нуждалась, Георг учился сначала в гимназии, а потом в университете. Сначала он преподавал математику в одной из частных школ Швейцарии. Физикой Георг Ом стал интересоваться позже. Свою научную деятельность начал с ремонта приборов и изучения научной литературы. Создание первого гальванического элемента открыло перед физиками новую область исследований, и Ом сделал важнейший шаг на пути создания теории электрических цепей. В 1825 году он представил научному миру плоды своего труда в виде статьи, которую озаглавил “Предварительное сообщение о законе, по которому металлы проводят электричество”. Сейчас это сообщение мы называем законом его имени. В честь этого ученого также названа единица сопротивления.

      Цель урока: обобщить знания учащихся об электрическом токе и напряжении и установить на опыте зависимость силы тока от напряжения на однородном участке электрической цепи и от сопротивления этого участка, вывести закон Ома для участка цепи.

      Задачи урока:

      • Образовательные : закрепить понятия сила тока, напряжение, сопротивление; опытным путем установить зависимость силы тока от напряжения и сопротивления; научить учащихся, используя закон Ома решать расчетные задачи.
      • Развивающие : развивать умения наблюдать, сопоставлять, сравнивать и обобщать и делать выводы из опытных фактов; формировать навыки культуры проведения физического эксперимента.
      • Воспитательные : развивать познавательный интерес к предмету; приучать учащихся к аккуратности при оформлении решений задач; прививать умения организовывать свою работу в определённом промежутке времени, доброжелательному общению, взаимопомощи, взаимопроверке и самооценке.

      Тип урока: урок формирования новых знаний.

      Оборудование к уроку: амперметр, вольтметр, источник тока, магазин сопротивлений, ключ, соединительные провода, Презентация «Закон Ома для участка цепи», мультимедийный проектор, компьютер, экран.

      План урока

      І. Организационный момент.
      ІІ. Проверка домашнего задания.
      ІІІ. Актуализация знаний.
      ІV. Изучение нового материала.
      V. Физминутка.
      VІ. Закрепление изученного материала.
      VІІ. Домашнее задание.
      VІІІ. Подведение итогов урока, оценка работ учащихся.
      ІХ. Рефлексия.

      ХОД УРОКА

      І .Организационный момент (самоопределение к деятельности)

      Цель: проверить готовность обучающихся, их настрой на работу.

      Учитель: Здравствуйте, ребята! Я рада вас видеть на уроке! Посмотрите друг на друга. Улыбнитесь, пошлите друг другу положительные эмоции! Перед вами лежат три карточки с нарисованными на них смайликами. Выберите ту карточку, которая соответствует вашему настроению в данный момент. У вас на столе лежат оценочные листки (Приложение 1 ), куда вы будете вносить оценки за все ваши действия, а в конце выставите итоговую оценку за урок.

      ІІ. Подготовка к восприятию нового материала

      Цель: подвести учащихся к формулировке цели урока.
      На предыдущих уроках мы с вами изучили, что каждая электрическая цепь характеризуется тремя физическими величинами. Давайте вспомним, какими?

      Ученик: Сила тока, напряжение, и сопротивление.

      Учитель: Дайте небольшую характеристику каждой из этих величин, по плану:

      • Назвать величину.
      • Что характеризует данная величина?;
      • Как обозначается?
      • В каких единицах измеряется?
      • Каким прибором измеряется?
      • По какой формуле вычисляется?

      Напряжение

      Сила тока

      Сопротивление

      Напряжение характеризует электрическое поле в проводнике – «пастух»Характеризует электрический ток в проводнике –
      какой заряд, то есть сколько электронов пройдут
      по проводнику за 1 с
      Характеризует сам проводник (или цепь)
      Обозначается буквой U Обозначается буквой I Обозначается буквой R
      Единица напряжения 1 ВЕдиница напряжения 1 АЕдиница измерения 1 Ом
      Измеряется вольтметромИзмеряется амперметромИзмеряется омметром
      Вычисляется по формуле:Вычисляется по формуле:Вычисляется по формуле:

      Эти физические величины мы изучали по отдельности, но ведь они существуют и характеризуют нечто общее – электрическую цепь. Значит, они должны быть связаны между собой. На прошлом уроке мы установили зависимость между силой тока и напряжением. Какая это зависимость?

      Ученик: Чем больше напряжение, тем больше сила тока, и наоборот: чем меньше напряжение, тем меньше сила тока.

      Учитель: А как называется такая зависимость?

      Ученик: Прямая зависимость!

      Учитель: Графиком этой зависимости будет прямая! Мы установили зависимость между силой тока и напряжением, но у нас еще есть третья величина – сопротивление. И мы не знаем, как связаны эти величины. Как вы думаете, какова цель нашего сегодняшнего урока?

      Ученики: Выяснить зависимость между тремя величинами: силой тока, напряжением и сопротивлением.

      Учитель: Цель урока мы с вами поставили. И эту зависимость мы будем искать опытным путем.

      ІІІ. Актуализация опорных знаний (фронтальная работа с классом)

      Цель: подвести учащихся к формулировке темы урока.

      Учитель: Чтобы узнать тему нашего сегодняшнего урока, необходимо разгадать кроссворд (Приложение 2 ) и отгадать выделенное слово по вертикали. (Каждый выполняет эту работу самостоятельно, а потом мы проверяем).

      Вопросы к кроссворду:

      • Бывает положительным, бывает отрицательным. (Заряд)
      • Как включают вольтметр в цепь? (Параллельно)
      • Единица измерения электрического заряда (количества электричества) в Международной системе единиц (СИ). (Кулон)
      • Упорядоченное движение заряженных частиц. (Ток)
      • Физическая величина, характеризующая электрическое поле, которое создаёт ток. (Напряжение)
      • Единица напряжения. (Вольт)
      • Прибор для измерения напряжения. (Вольтметр)
      • Прибор для измерения силы тока. (Амперметр)

      Учитель: Какое выражение мы получили?

      Ученики: Закон Ома.

      Учитель: Тема нашего сегодняшнего урока – Закон Ома. Откройте тетради и запишите
      тему урока: «Закон Ома для участка цепи».

      ІV. Изучение нового материала (работа в группах)

      Цель: Выяснить экспериментальным путем зависимость силы тока на участке цепи от сопротивления проводника.
      Чтобы рассмотреть зависимость между силой тока, напряжением и сопротивлением, нужно обратиться к опыту. Немецкий ученый – философ И. Кант сказал так: “Все наше знание начинается с опыта”.
      Разделимся на 2 группы. Первая группа выяснит, как зависит сила тока от напряжения на участке цепи при постоянном сопротивлении этого участка, вторая – как сила тока зависит от сопротивления проводника, при постоянном напряжении на его концах. А затем мы совместно сделаем общий вывод о том, как зависит сила тока одновременно от напряжения и сопротивления, т.е. решим основную задачу урока.
      На столах у вас есть все необходимое оборудование, а также схемы, инструкции по выполнению эксперимента и таблицы, которые необходимо заполнить (Приложение 3 ).
      Техника безопасности при работе с электроприборами:

      • На рабочем месте провода располагайте аккуратно, плотно соединяйте клеммы с приборами.
      • После сборки всей электрической цепи, не включайте до тех пор, пока всё не проверит учитель.
      • Все изменения в электрической цепи можно проводить только при выключенном источнике электропитания.
      • По окончании работ отключите источник электропитания и разберите электрическую цепь.

      1 группа

      U, BI, AR, Ом
      1
      1
      1

      2 группа

      Инструкция по выполнению исследования

      U, BI, AR, Ом
      const
      const
      const
      const

      Внимательно следите за правильностью подключения измерительных приборов!

      Учитель: Послушаем выводы 1 группы.

      Учащиеся: С увеличением напряжения сила тока в проводнике возрастает при постоянном сопротивлении, т.е. при R = const, I ~ U .

      Учитель: Послушаем выводы 2 группы.

      Учащиеся: С увеличением сопротивления проводника сила тока уменьшается, т.е. при U = const, I ~ 1/R.

      Учитель: Тогда сможем записать:

      Мы получили математическую запись закона Ома, который читается так: “Сила тока на участке цепи прямо пропорциональна напряжению на концах этого участка и обратно пропорциональна его сопротивлению”. Данный закон немецкий физик Георг Ом открыл в 1827 году.

      Учащийся: Историческая справка (доклад ученика) (Приложение 4 )

      Учитель: Для запоминания формулы закона Ома и последующего его применения для решения задач лучше пользоваться треугольником.
      Графическая зависимость силы тока от напряжения называется ВАХ (вольт – амперная характеристика) проводника.

      Этот закон является основным в электротехнике, радиотехнике, в работе всех электрических устройств. Не знаешь закона Ома – сиди дома! Применяя основной закон электрической цепи (закон Ома), можно объяснить многие природные явления, которые на первый взгляд кажутся загадочными и парадоксальными. Например, всем известно, что любой контакт человека с электрическими проводами, находящимися под напряжением, является смертельно опасным. Всего лишь одно прикосновение к оборвавшемуся проводу высоковольтной линии способно убить электрическим током человека или животное. Но в то же время, мы постоянно видим, как птицы спокойно усаживаются на высоковольтные провода электропередач, и ничто не угрожает жизни этих живых существ. Тогда как же найти объяснение такому парадоксу?
      А объясняется подобное явление довольно просто, если представить, что находящаяся на электрическом проводе птица – это один из участков электрической цепи, сопротивление которого значительно превышает сопротивление другого участка той же цепи (то есть небольшого промежутка между лапками птицы). Следовательно, сила электрического тока, воздействующая на первый участок цепи, то есть на тело птицы, будет совершенно безопасной для неё.
      Однако полная безопасность гарантирована ей только при соприкосновении с участком высоковольтного провода. Но стоит только птице, усевшейся на линию электропередач, задеть крылом или клювом провод или какой-либо предмет, находящийся вблизи от провода (например, телеграфный столб), то птица неминуемо погибнет. Ведь столб непосредственно связан с землёй, и поток электрических зарядов, переходя на тело птицы, способен мгновенно убить её, стремительно двигаясь по направлению к земле. К сожалению, по этой причине в городах гибнет немало птиц.
      Для защиты пернатых от губительного воздействия электричества зарубежными учеными были разработаны специальные устройства – насесты для птиц, изолированные от электрического тока. Такие приспособления размещали на высоковольтных линиях электропередач. Птицы, усаживаясь на изолированный насест, могут без всякого риска для жизни прикасаться клювом, крыльями или хвостом к проводам, столбам или кронштейнам.

      V. Физминутка

      Цель: Сохранение здоровья школьников, поддержание тонуса.

      VІ . Закрепление изученного материала

      Цель: проверка уровня усвоения материала и умения применять на практике.

      Учитель:

      а) Вернемся к закону, который мы получили, и посмотрим, как его можно применять для расчета одной величины, зная две другие.

      Познакомимся с классификацией задач по теме «Закон Ома»:

      б) Решение задач.

      I. Логические задачи на понимание взаимосвязи между током, напряжением и сопротивлением цепи

      (Ответ: так как сила тока в цепи прямо пропорциональна напряжению в ней, то при увеличении напряжения в 4 раза и сила тока увеличится в 4 раза (при неизменном сопротивлении цепи)).

      2. Сопротивление цепи увеличили в 2 раза. Как изменится сила тока, если напряжение в цепи останется неизменным?

      (Ответ: так как сила тока в цепи обратно пропорциональна сопротивлению цепи, то при увеличении сопротивления в 2 раза сила тока уменьшится в 2 раза (при неизменном напряжении)).

      II. Расчетные задачи на применение закона Ома

      1. Напряжение на зажимах электрического утюга 220 В, сопротивление нагревательного элемента (спирали внутри корпуса) равно 50 Ом. Чему равна сила тока в нагревательном элементе?

      2. Сила тока в спирали электрической лампы 0,7 А, сопротивление лампы 310 Ом. Определить напряжение, под которым находится лампа.

      3. Сила тока в спирали электрической плитки равна 5 А. Напряжение, под которым находится плитка, равно 200 В. Определить сопротивление спирали.

      III. Задача-график

      Пользуясь графиком зависимости силы тока от напряжения между его концами, определить сопротивление этого проводника.

      VІІ. Домашнее задание: § 44, упр.29 (2,3,4).

      Инструктаж по выполнению домашнего задания.

      VІІІ. Подведение итогов урока

      Цель: Соотнесение поставленных целей достигнутым результатам.

      Сегодня на уроке вы познакомились с одним из важных законов при изучении электрических явлений “Закон Ома для участка цепи”. Научились устанавливать зависимость физических величин путем проведения эксперимента, решения задач.
      1. Между какими величинами устанавливает зависимость закон Ома?
      2. В какой формуле выражена эта взаимозависимость?
      3.Что понравилось на уроке?
      4. Какие задания вам показались наиболее интересными? Трудными? Важными?

      – А сейчас поставьте итоговое количество баллов в свой оценочный лист. Согласно, него, я выставлю оценку за этот урок.

      ІХ. Рефлексия (показать картинку со смайликом).

      • На уроке было комфортно и все понятно.
      • На уроке немного затруднялся, не все понятно.
      • На уроке было трудно, ничего не понял.

      Заряд

      Сила тока

      Время

      Напряжение

      Работа


      Проверь себя!

      U, Ом

      4. Формула силы тока

      U, Ом

      U, В

      R, Ом

      5. Формула сопротивления

      I, В

      6. Формула напряжения

      R= SL/ ρ

      U, В

      R= ρ L/s

      R= S ρ /L


      Соотнеси величины и единицы их измерения, обозначения

      Заряд

      Сила тока

      Время

      Напряжение

      Работа


      Проверь себя!

      1. Обозначение силы тока, единица измерения

      2. Обозначение сопротивления, единица измерения

      3. Обозначение напряжения, единица измерения

      U, Ом

      4. Формула силы тока

      U, Ом

      U, В

      R, Ом

      5. Формула сопротивления

      I, В

      6. Формула напряжения

      U, В


      Цель урока:

      • установить зависимость между силой тока, напряжением на участке цепи и сопротивлением этого участка.



      Первая серия опытов

      • Устанавливается зависимость I от U
      • R остается неизменной (R=2Ом)

      Сделайте вывод

      Зависимость прямо пропорциональная


      Вторая серия опытов

      • Устанавливается зависимость I от R
      • U остается неизменным U=4 В

      Сделайте вывод

      Зависимость обратно пропорциональная

      0 1 2 3 4 5 6


      Формулировка:

      Сила тока в участке цепи прямо пропорциональна напряжению на концах этого участка и обратно пропорциональна его сопротивлению


      Математическая запись закона:


      Магический треугольник:


      Ом вырвал у природы так долго

      скрываемую тайну и передал ее

      в руки современников»

      Ломмель Эуген Корнелиус Йозеф


      Какую силу тока показывает амперметр А3 ?


      Примеры решения задач

      • Определите силу тока в электрочайнике, если он включен в сеть с напряжением 220 В, а сопротивление нити накала при работе чайника 40 Ом.

      I=220В/40 Ом=5,5А

      Ответ: 5.5 А


      Чему равно сопротивление лампочки?


      Решим задачу

      На рисунке изображены графики зависимости силы тока от напряжения для двух проводников А и В. Какой из этих проводников обладает большим сопротивлением?


      II вариант

      I вариант

      1. По данным приведенным на

      рисунке определите показания

      вольтметра.

      1. По данным приведенным на

      рисунке определите показания

      амперметра.

      2. Лампа рассчитана на

      напряжение 6В и силу тока 4 А.

      Каково сопротивление лампы?

      2. Лампа рассчитана на

      напряжение 127 В, имеет

      сопротивление 254 Ом.

      Вычислите силу тока в лампе.

      • U = 110 B;
      • I = 0,5 A.
      • I = 3A;
      • R = 1,5 Ом.

      ПОДВЕДЕМ ИТОГИ .

      СФОРМУЛИРУЙТЕ ЗАКОН ОМА.

      КАК ИЗМЕНИТСЯ СИЛА ТОКА НА УЧАСТКЕ ЦЕПИ, ЕСЛИ ПРИ НЕИЗМЕННОМ СОПРОТИВЛЕНИИ УВЕЛИЧИВАТЬ НАПРЯЖЕНИЕ НА ЕГО КОНЦАХ?

      КАК ИЗМЕНИТСЯ СИЛА ТОКА, ЕСЛИ ПРИ НЕИЗМЕННОМ НАПРЯЖЕНИИ УВЕЛИЧИТЬ СОПРОТИВЛЕНИЕ УЧАСТКА ЦЕПИ?

      КАК С ПОМОЩЬЮ АМПЕРМЕТРА И ВОЛЬТМЕТРА МОЖНО ИЗМЕРИТЬ СОПРОТИВЛЕНИЕ ПРОВОДНИКА?

      Домашнее задание:

      § § 42,44. упр. 17(2),19 (4,6)

      Подготовить сообщение с презентацией «Георг Ом», «Роль электричества в современной жизни»


      Когда немецкий электротехник Георг Симон Ом положил на стол ректора Берлинского университета свою диссертацию, где впервые был сформулирован этот закон, без которого невозможен ни один электротехнический расчет, он получил весьма резкую резолюцию. В ней говорилось, что электричество не поддается никакому математическом описанию, так как электричество – это собственный гнев, собственное бушевание тела, его гневное Я, которое проявляется в каждом теле, когда его раздражают. Ректором Берлинского университета был в те годы Георг Вильгельм Фридрих Гегель.


      Родился в Эрлангере, в семье бедного слесаря. Мать Георга – Мария Елизавет, умерла при родах, когда мальчику исполнилось десять лет. Отец его – Иоганн Вольфганг, весьма развитой и образованный человек, с детства внушал сыну любовь к математике и физике, и поместил его в гимназию, которая курировалась университетом; по окончании курса в 1806 г. Наиболее известные работы Ома касались вопросов о прохождении электрического тока и привели к знаменитому «закону Ома», связывающему сопротивление цепи гальванического тока, электродвижущей в нём силы и силы тока, и лежащему в основе всего современного учения об электричестве.

      Лекция № 6 – Закон Ома

              

      Георг Симон Ом

          Закон Ома, основанный на опытах, представляет собой в электротехнике основной закон, который устанавливает связь силы электрического тока с сопротивлением и напряжением.

         Закон Ома – полученный экспериментальным путём (эмпирический) закон, который устанавливает связь силы тока в проводнике с напряжением на концах проводника и его сопротивлением, был открыт в 1826 году немецким физиком-экспериментатором Георгом Омом.

         

      Закон Ома для участка цепи

       

         Строгая формулировка закона Ома может быть записана так:

            сила тока в проводнике прямо пропорциональна напряжению на его концах (разности потенциалов) и обратно пропорциональна сопротивлению этого проводника.

         Формула закона Ома записывается в следующем виде:

      где

      I – сила тока в проводнике, единица измерения силы тока – ампер [А];

      U – электрическое напряжение (разность потенциалов), единица измерения напряжения- вольт [В];

      R – электрическое сопротивление проводника, единица измерения электрического сопротивления – ом [Ом].

          Согласно закону Ома, увеличение напряжения, например, в два раза при фиксированном сопротивлении проводника, приведёт к увеличению силы тока также в два раза

          И напротив, уменьшение тока в два раза при фиксированном напряжении будет означать, что сопротивление увеличилось в два раза.

          Рассмотрим простейший случай применения закона Ома.

          Пусть дан некоторый проводник сопротивлением 3 Ом под напряжением 12 В. Тогда, по определению закона Ома, по данному проводнику течет ток равный:

            Существует мнемоническое правило для запоминания этого закона, которое можно назвать треугольник Ома. Изобразим все три характеристики (напряжение, сила тока и сопротивление) в виде треугольника. В вершине которого находится напряжение, в нижней левой части – сила тока, а в правой – сопротивление.

      Правило работы такое: закрываем пальцем величину в треугольнике, которую нужно найти, тогда две оставшиеся дадут верную формулу для поиска закрытой.

      Где и когда можно применять закон Ома?

             Закон Ома в упомянутой форме справедлив в достаточно широких пределах для металлов. Он выполняется до тех пор, пока металл не начнет плавиться. Менее широкий диапазон применения у растворов (расплавов) электролитов и в сильно ионизированных газах (плазме).

          Работая с электрическими схемами, иногда требуется определять падение напряжения на определенном элементе. Если это будет резистор с известной величиной сопротивления (она проставляется на корпусе), а также известен проходящий через него ток, узнать напряжение можно с помощью формулы Ома, не подключая вольтметр.

      Значение Закона Ома

           Закон Ома определяет силу тока в электрической цепи при заданном напряжении и известном сопротивлении. Он позволяет рассчитать тепловые, химические и магнитные действия тока, так как они зависят от силы тока.

         Закон Ома является чрезвычайно полезным в технике(электронной/электрической), поскольку он касается трех основных электрических величин: тока, напряжения и сопротивления. Он показывает, как эти три величины являются взаимозависимыми на макроскопическом уровне.

          Если бы было можно охарактеризовать закон Ома простыми словами, то наглядно это выглядело бы так:

             Из закона Ома вытекает, что замыкать обычную осветительную сеть проводником малого сопротивления опасно. Сила тока окажется настолько большой, что это может иметь тяжелые последствия.

       

      Задача 1.1

        Рассчитать силу тока, проходящую по медному проводу длиной 100 м, площадью поперечного сечения 0,5 мм2, если к концам провода приложено напряжение 12 B.

       

        Задачка простая, заключается в нахождении сопротивления медной проволоки с последующим расчетом силы тока по формуле закона Ома для участка цепи. Приступим.

       

      Закон Ома для полной цепи 

        Формулировка закона Ома для полной цепи – сила тока прямо пропорциональна сумме ЭДС цепи, и обратно пропорциональна сумме сопротивлений источника и цепи , где E – ЭДС, R- сопротивление цепи, r – внутреннее сопротивление источника.

        Здесь могут возникнуть вопросы. Например, что такое ЭДС?

          Электродвижущая сила – это физическая величина, которая характеризует работу внешних сил в источнике ЭДС. К примеру, в обычной пальчиковой батарейке, ЭДС является химическая реакция, которая заставляет перемещаться заряды от одного полюса к другому. Само слово электродвижущая говорит о том, что эта сила двигает  заряд.

        В каждом источнике присутствует внутреннее сопротивление r, оно зависит от параметров самого источника. В цепи также существует сопротивление R, оно зависит от параметров самой цепи.

        Формулу закона Ома для полной цепи можно представить в другом виде. А именно: ЭДС источника цепи равна сумме падений напряжения на источнике и на внешней цепи.

        

      Для закрепления материала, решим две задачи на формулу закона Ома для полной цепи.

      Задача 2.1

        Найти силу тока в цепи, если известно что сопротивление цепи 11 Ом, а источник подключенный к ней имеет ЭДС 12 В и внутреннее сопротивление 1 Ом.

       

        Теперь решим задачу посложнее.

       Задача 2.2

        Источник ЭДС подключен к резистору сопротивлением 10 Ом с помощью медного провода длиной 1 м и площадью поперечного сечения 1 мм2. Найти силу тока, зная что ЭДС источника равно 12 В, а внутреннее сопротивление 1,9825 Ом.

      Приступим.

       

       

      Закон ома для участка сети

      Закон Ома, основанный на опытах, представляет собой в электротехнике основной закон, который устанавливает связь силы электрического тока с сопротивлением и напряжением.

      Появление смартфонов, гаджетов, бытовых приборов и прочей электротехники коренным образом изменило облик современного человека. Приложены огромные усилия, направленные на исследование физических закономерностей для улучшения старой и создания новой техники. Одной из таких зависимостей является закон Ома.

      Закон Ома – полученный экспериментальным путём (эмпирический) закон, который устанавливает связь силы тока в проводнике с напряжением на концах проводника и его сопротивлением, был открыт в 1826 году немецким физиком-экспериментатором Георгом Омом.

      Строгая формулировка закона Ома может быть записана так: сила тока в проводнике прямо пропорциональна напряжению на его концах (разности потенциалов) и обратно пропорциональна сопротивлению этого проводника.

      Формула закона Ома записывается в следующем виде:

      U – электрическое напряжение (разность потенциалов), единица измерения напряжения- вольт [В];

      Согласно закону Ома, увеличение напряжения, например, в два раза при фиксированном сопротивлении проводника, приведёт к увеличению силы тока также в два раза

      И напротив, уменьшение тока в два раза при фиксированном напряжении будет означать, что сопротивление увеличилось в два раза.

      Рассмотрим простейший случай применения закона Ома. Пусть дан некоторый проводник сопротивлением 3 Ом под напряжением 12 В. Тогда, по определению закона Ома, по данному проводнику течет ток равный:

      Существует мнемоническое правило для запоминания этого закона, которое можно назвать треугольник Ома. Изобразим все три характеристики (напряжение, сила тока и сопротивление) в виде треугольника. В вершине которого находится напряжение, в нижней левой части – сила тока, а в правой – сопротивление.

      Правило работы такое: закрываем пальцем величину в треугольнике, которую нужно найти, тогда две оставшиеся дадут верную формулу для поиска закрытой.

      Где и когда можно применять закон Ома?

      Закон Ома в упомянутой форме справедлив в достаточно широких пределах для металлов. Он выполняется до тех пор, пока металл не начнет плавиться. Менее широкий диапазон применения у растворов (расплавов) электролитов и в сильно ионизированных газах (плазме).

      Работая с электрическими схемами, иногда требуется определять падение напряжения на определенном элементе. Если это будет резистор с известной величиной сопротивления (она проставляется на корпусе), а также известен проходящий через него ток, узнать напряжение можно с помощью формулы Ома, не подключая вольтметр.

      Значение Закона Ома

      Закон Ома определяет силу тока в электрической цепи при заданном напряжении и известном сопротивлении.

      Он позволяет рассчитать тепловые, химические и магнитные действия тока, так как они зависят от силы тока.

      Закон Ома является чрезвычайно полезным в технике(электронной/электрической), поскольку он касается трех основных электрических величин: тока, напряжения и сопротивления. Он показывает, как эти три величины являются взаимозависимыми на макроскопическом уровне.

      Если бы было можно охарактеризовать закон Ома простыми словами, то наглядно это выглядело бы так:

      Из закона Ома вытекает, что замыкать обычную осветительную сеть проводником малого сопротивления опасно. Сила тока окажется настолько большой, что это может иметь тяжелые последствия.

      Немецкий физик Георг Симон Ом (1787—1854) открыл основной закон электрической цепи.

      Закон Ома для участка цепи:

      Определение: Cила тока I на участке электрической цепи прямо пропорциональна напряжению U на концах участка и обратно пропорциональна его сопротивлению R.

      1. I — сила тока (в системе СИ измеряется — Ампер)
        • Сила тока в проводнике прямо пропорциональна напряжению и обратно пропорциональна сопротивлению.
        • Формула: I=frac
        • U — напряжение (в системе СИ измеряется — Вольт)

        Падение напряжения на участке проводника равно произведению силы тока в проводнике на сопротивление этого участка.

        Формула: U=IR

      2. R— электрическое сопротивление (в системе СИ измеряется — Ом).
        • Электрическое сопротивление R это отношение напряжения на концах проводника к силе тока, текущего по проводнику.
        • Формула R=frac
        • Определение единицы сопротивления — Ом

          1 Ом представляет собой электрическое сопротивление участка проводника, по которому при напряжении 1(Вольт) протекает ток 1 (Ампер).

          Закон Ома для полной цепи

          Определение: Сила тока в цепи пропорциональна действующей в цепи ЭДС и обратно пропорциональна сумме сопротивлений цепи и внутреннего сопротивления источника

          Формула I=frac <varepsilon>

          • varepsilon — ЭДС источника напряжения, В;
          • I — сила тока в цепи, А;
          • R — сопротивление всех внешних элементов цепи, Ом;
          • r — внутреннее сопротивление источника напряжения, Ом.

          Как запомнить формулы закона Ома

          Треугольник Ома поможет запомнить закон. Нужно закрыть искомую величину, и два других символа дадут формулу для её вычисления.

          .

          • U — электрическое напряжение;
          • I — сила тока;
          • P — электрическая мощность;
          • R — электрическое сопротивление

          Смотри также:

          Для закрепления своих знаний решай задания и варианты ЕГЭ по физике с ответами и пояснениями.

          Закон Ома — физический закон, определяющий зависимость между электрическими величинами — напряжением, сопротивлением и током для проводников.
          Впервые открыл и описал его в 1826 году немецкий физик Георг Ом, показавший (с помощью гальванометра) количественную связь между электродвижущей силой, электрическим током и свойствами проводника, как пропорциональную зависимость.
          Впоследствии свойства проводника, способные противостоять электрическому току на основе этой зависимости, стали называть электрическим сопротивлением (Resistance), обозначать в расчётах и на схемах буквой R и измерять в Омах в честь первооткрывателя.
          Сам источник электрической энергии также обладает внутренним сопротивлением, которое принято обозначать буквой r.

          Закон Ома для участка цепи

          Со школьного курса физики всем хорошо известна классическая трактовка Закона Ома:

          Сила тока в проводнике прямо пропорциональна напряжению на концах проводника и обратно пропорциональна его сопротивлению.

          Это значит, если к концам проводника сопротивлением R = 1 Ом приложено напряжение U = 1 Вольт, тогда величина тока I в проводнике будет равна 1/1 = 1 Ампер.

          Отсюда следуют ещё два полезных соотношения:

          Если в проводнике, сопротивлением 1 Ом, протекает ток 1 Ампер, значит на концах проводника напряжение 1 Вольт (падение напряжения).

          Если на концах проводника есть напряжение 1 Вольт и по нему протекает ток 1 Ампер, значит сопротивление проводника равно 1 Ом.

          Вышеописанные формулы в таком виде могут быть применимы для переменного тока лишь в том случае, если цепь состоит только из активного сопротивления R.
          Кроме того, следует помнить, что Закон Ома справедлив только для линейных элементов цепи.

          Предлагается простой Онлайн-калькулятор для практических расчётов.

          Закон Ома. Расчёт напряжения, сопротивления, тока, мощности.
          После сброса ввести два любых известных параметра.

          Закон Ома для замкнутой цепи

          Если к источнику питания подключить внешнюю цепь сопротивлением R, в цепи пойдёт ток с учётом внутреннего сопротивления источника:

          I — Сила тока в цепи.
          — Электродвижущая сила (ЭДС) — величина напряжения источника питания не зависящая от внешней цепи (без нагрузки). Характеризуется потенциальной энергией источника.
          r — Внутреннее сопротивление источника питания.

          Для электродвижущей силы внешнеее сопротивление R и внутреннее r соединены последовательно, значит величина тока в цепи определится значением ЭДС и суммой сопротивлений: I = /(R+r) .

          Напряжение на выводах внешней цепи определится исходя из силы тока и сопротивления R соотношением, которое уже рассматривалось выше: U = IR.
          Напряжение U, при подключении нагрузки R, всегда будет меньше чем ЭДС на величину произведения I*r, которую называют падением напряжения на внутреннем сопротивлении источника питания.
          С этим явлением мы сталкиваемся достаточно часто, когда видим в работе частично разряженные батарейки или аккумуляторы.
          По мере разряда, увеличивается их внутреннее сопротивление, следовательно, увеличивается падение напряжение внутри источника, значит уменьшается внешнее напряжение U = — I*r.
          Чем меньше ток и внутреннее сопротивление источника, тем ближе по значению его ЭДС и напряжение на его выводах U.
          Если ток в цепи равен нулю, следовательно, = U. Цепь разомкнута, ЭДС источника равна напряжению на его выводах.

          В случаях, когда внутренним сопротивлением источника можно пренебречь (r ≈ 0), напряжение на выводах источника будет равно ЭДС ( ≈ U ) независимо от сопротивления внешней цепи R.
          Такой источник питания называют источником напряжения.

          Закон Ома для переменного тока

          При наличии индуктивности или ёмкости в цепи переменного тока необходимо учитывать их реактивное сопротивление.
          В таком случае запись Закона Ома будет иметь вид:

          Здесь Z — полное (комплексное) сопротивление цепи — импеданс. В него входит активная R и реактивная X составляющие.
          Реактивное сопротивление зависит от номиналов реактивных элементов, от частоты и формы тока в цепи.
          Более подробно ознакомится с комплексным сопротивлением можно на страничке импеданс.

          С учётом сдвига фаз φ, созданного реактивными элементами, для синусоидального переменного тока обычно записывают Закон Ома в комплексной форме:

          — комплексная амплитуда тока. = Iampe jφ
          — комплексная амплитуда напряжения. = Uampe jφ
          — комплексное сопротивление. Импеданс.
          φ — угол сдвига фаз между током и напряжением.
          e — константа, основание натурального логарифма.
          j — мнимая единица.
          Iamp , Uamp — амплитудные значения синусоидального тока и напряжения.

          Нелинейные элементы и цепи

          Закон Ома не является фундаментальным законом природы и может быть применим в ограниченных случаях, например, для большинства проводников.
          Его невозможно использовать для расчёта напряжения и тока в полупроводниковых или электровакуумных приборах, где эта зависимость не является пропорциональной и её можно определять только с помощью вольтамперной характеристики (ВАХ). К данной категории элементов относятся все полупроводниковые приборы (диоды, транзисторы, стабилитроны, тиристоры, варикапы и т.д.) и электронные лампы.
          Такие элементы и цепи, в которых они используются, называют нелинейными.

          Замечания и предложения принимаются и приветствуются!

          Закон Ома | Клуб электроники

          Закон Ома | Клуб электроники

          Следующая страница: Мощность и энергия

          См. также: Напряжение и ток | Сопротивление

          Закон Ома показывает взаимосвязь между напряжением, током и сопротивлением.

          Чтобы ток протекал через сопротивление, на этом сопротивлении должно быть напряжение. Закон Ома показывает взаимосвязь между тремя величинами: напряжением, током и сопротивлением.

          Закон Ома можно записать в виде словесного уравнения :

          напряжение = ток × сопротивление

          Или с помощью символов для обозначения величин напряжения (В), тока (I) и сопротивления (R):

          4 × R

          На самом деле это может быть записано тремя способами, и вы можете выбрать вариант, который лучше всего подходит для вашей цели:

          В = I × R
          01
          I = В
          R
          R =  V
           I

          The VIR triangle – a way to remember Ohm’s Law

             V
           И    Р

          Вы можете использовать треугольник VIR, чтобы запомнить три варианта закона Ома.

          • Чтобы рассчитать напряжение , В : наведите палец на V, это оставляет I R, поэтому уравнение V = I × R 90 122
          • Чтобы рассчитать ток , I : положите палец на I, это оставляет V над R, поэтому уравнение I =  V / R
          • Чтобы рассчитать сопротивление , R : поместите палец на R, это оставляет V над I, поэтому уравнение R =  В / И

          Используйте правильные единицы измерения

          Для большинства электронных схем ток слишком велик, а сопротивление слишком мало, поэтому мы часто измеряем ток в миллиамперах (мА) и сопротивление в килоомах (к).

          1 мА = 0,001 А
          1 к = 1000 .

          Уравнения закона Ома работают, если вы используете V, A и , или если вы используете В, мА и к.

          Крайне важно использовать правильные единицы измерения для трех величин в законе Ома, иначе расчеты дадут неверные значения.

          Вы можете использовать любой из этих двух наборов единиц измерения:

          V = напряжение в вольтах (В)
          I = ток в амперах (А)
          R = сопротивление в омах ()

          или

          V = напряжение в вольтах (В)
          I = ток в миллиамперах (мА)
          R = сопротивление в кОм (к)

          Вы не должны смешивать эти наборы единиц измерения в уравнениях, поэтому вам может потребоваться преобразование между мА и А или k и .

          Преобразование единиц

          Преобразовать единицы при необходимости, используя эти правила:

          для тока:
          1MA = 0,001A
          1781178
          1A = 1000MA
          1A = 1000MA
          . 1000
          1 = 0,001k
          Пример преобразования единиц измерения0003

          470 = 0,47k

          3,3k = 3300



          Расчеты по закону Ома

          Используйте этот метод для проведения расчетов:

          1. Запишите значения , при необходимости переведя единицы измерения.
          2. Выберите нужное уравнение (используйте треугольник VIR).
          3. Подставьте чисел в уравнение и вычислите ответ.

          Должно быть V ery E asy N ой! См. примеры ниже:

          Пример 1:

          3 В подается на резистор 6, каков ток?

          • V значения: V = 3V, I = ?, R = 6
          • E предложение: I = V / R
          • N цифры: Ток, I = 3 / 6 = 0,5 А
          Пример 2:

          Лампа, подключенная к батарее 6 В, пропускает ток силой 60 мА. Каково сопротивление лампы?

          • В значения: V = 6 В, I = 60 мА, R = ?
          • E предложение: R = V / I
          • N цифры: Сопротивление, R = 6 / 60 = 0,1k = 100 
            (использование мА для тока означает, что расчет дает сопротивление в кОм)
          Пример 3:

          Резистор 1,2 кОм пропускает ток 0,2 А, каково напряжение на нем?

          • В значения: В = ?, I = 0,2А, R = 1,2к = 1200 90 146 (1,2 КБ преобразуется в 1200 т.к. А и k нельзя использовать вместе)
          • E формула: V = I × R
          • N цифры: V = 0,2 × 1200 = 240 В
          Пример 4:

          9 В подается на резистор 15 кОм, какова сила тока?

          • В значения: V = 9В, I = ?, R = 15k
          • E предложение: I = V / R
          • N цифры: Ток, I = 9 / 15 = 0,6 мА
            (использование k для сопротивления означает, что расчет дает ток в мА)

          Следующая страница: Энергия и энергия | Исследование


          Закон

          Ом – Как соотносятся напряжение, ток и сопротивление | Закон Ома

          Первое и, возможно, самое важное соотношение между током, напряжением и сопротивлением называется законом Ома. Он был открыт Георгом Симоном Омом и опубликован в его статье 1827 года «Математическое исследование гальванической цепи».

          Напряжение, ток и сопротивление

          Электрическая цепь образуется, когда создается токопроводящий путь, позволяющий электрическому заряду непрерывно двигаться. Это непрерывное движение электрического заряда по проводникам цепи называется или , и его часто называют «потоком», точно так же, как течение жидкости через полую трубу.

          Сила, побуждающая носители заряда «течь» по цепи, называется напряжением . Напряжение — это особая мера потенциальной энергии, которая всегда является относительной между двумя точками.

          Когда мы говорим об определенной величине напряжения, присутствующего в цепи, мы имеем в виду измерение того, сколько потенциальной энергии существует для перемещения носителей заряда из одной конкретной точки этой цепи в другую конкретную точку. Без ссылки на две точки, термин «напряжение» не имеет значения.

          Ток имеет тенденцию проходить по проводникам с некоторой степенью трения или противодействия движению. Это противодействие движению правильнее назвать сопротивлением . Величина тока в цепи зависит от величины напряжения и величины сопротивления в цепи, препятствующего протеканию тока.

          Как и напряжение, сопротивление является величиной относительной между двумя точками. По этой причине величины напряжения и сопротивления часто указываются как находящиеся «между» или «поперек» двух точек цепи.

          Единицы измерения: вольт, ампер и ом

          Чтобы иметь возможность делать осмысленные утверждения об этих величинах в цепях, мы должны уметь описывать их величины таким же образом, как мы могли бы количественно определять массу, температуру, объем, длина или любая другая физическая величина. Для массы мы могли бы использовать единицы «килограмм» или «грамм».

          Для температуры мы можем использовать градусы Фаренгейта или градусы Цельсия. Вот стандартные единицы измерения электрического тока, напряжения и сопротивления:

           

           

          «Символ», данный для каждой величины, представляет собой стандартную букву алфавита, используемую для представления этой величины в алгебраическом уравнении. Подобные стандартные буквы распространены в физических и инженерных дисциплинах и признаны во всем мире.

          «Сокращенное обозначение единицы» для каждой величины представляет собой буквенный символ, используемый в качестве сокращенного обозначения для конкретной единицы измерения. И да, этот странно выглядящий символ «подкова» — заглавная греческая буква Ω, просто символ иностранный алфавит (приношу свои извинения всем греческим читателям).

          Каждая единица измерения названа в честь известного экспериментатора в области электричества: ампер в честь француза Андре М. Ампера, вольт в честь итальянца Алессандро Вольта и ом в честь немца Георга Симона Ома.

          Математический символ для каждой величины также имеет значение. «R» для сопротивления и «V» для напряжения говорят сами за себя, тогда как «I» для тока кажется немного странным. Считается, что «I» означает «интенсивность» (потока заряда), а другой символ напряжения, «E», означает «электродвижущая сила». Судя по тому исследованию, которое мне удалось провести, есть некоторые разногласия по поводу значения «я».

          Символы «E» и «V» по большей части взаимозаменяемы, хотя в некоторых текстах буква «E» используется для обозначения напряжения на источнике (например, аккумуляторе или генераторе), а «V» — для обозначения напряжения на чем-либо еще.

          Все эти символы обозначаются заглавными буквами, за исключением случаев, когда величина (особенно напряжение или ток) описывается в терминах короткого периода времени (называемого «мгновенным» значением). Например, напряжение батареи, стабильное в течение длительного периода времени, будет обозначаться заглавной буквой «Е», а пик напряжения удара молнии в тот момент, когда она попадает в линию электропередач, скорее всего, будет обозначаться символом со строчной буквой «e» (или строчной «v»), чтобы обозначить это значение как значение в один момент времени.

          То же соглашение о строчных буквах применимо и к текущему: строчная буква «i» представляет ток в некоторый момент времени. Однако большинство измерений постоянного тока (DC), будучи стабильными во времени, будут обозначены заглавными буквами.

          Кулон и электрический заряд

          Одной из основных единиц электрических измерений, которую часто преподают в начале курсов по электронике, но редко используют позже, является единица измерения кулона , которая является мерой электрического заряда, пропорционального количеству электронов в электронике. неуравновешенное состояние. Один кулон заряда равен 6 250 000 000 000 000 000 электронов.

          Символом количества электрического заряда является заглавная буква «Q», а единица измерения кулонов обозначается заглавной буквой «C». Так случилось, что единица измерения тока, ампер, равна 1 кулону заряда, проходящего через данную точку цепи за 1 секунду. В этих терминах ток — это скорость движения электрического заряда через проводник.

          Как указывалось ранее, напряжение является мерой потенциальной энергии на единицу заряда , доступной для мотивации протекания тока из одной точки в другую. Прежде чем мы сможем точно определить, что такое «вольт», мы должны понять, как измерить эту величину, которую мы называем «потенциальной энергией». Общей метрической единицей энергии любого вида является 9.0364 джоуля , равная работе, совершаемой силой в 1 ньютон при движении на 1 метр (в том же направлении).

          В имперских единицах это чуть меньше 3/4 фунта силы, приложенной на расстоянии 1 фута. Проще говоря, требуется около 1 джоуля энергии, чтобы поднять груз массой 3/4 фунта на 1 фут от земли или протащить что-либо на расстояние 1 фут, используя параллельную тяговую силу 3/4 фунта. В этих научных терминах 1 вольт равен 1 джоулю потенциальной электрической энергии на 1 кулон заряда (деленный на). Таким образом, 9Вольтовая батарея выделяет 9 джоулей энергии на каждый кулон заряда, пройденного по цепи.

          Эти единицы и символы для электрических величин станут очень важными для понимания, когда мы начнем исследовать отношения между ними в цепях.

           

          Уравнение закона Ома

          Основное открытие Ома заключалось в том, что количество электрического тока, протекающего через металлический проводник в цепи, прямо пропорционально приложенному к нему напряжению при любой заданной температуре. Ом выразил свое открытие в виде простого уравнения, описывающего взаимосвязь между напряжением, током и сопротивлением:

           

           

          В этом алгебраическом выражении напряжение (E) равно току (I), умноженному на сопротивление (R). Используя методы алгебры, мы можем преобразовать это уравнение в два варианта, находя I и R соответственно: :

           

           

          В приведенной выше схеме имеется только один источник напряжения (батарея слева) и только один источник сопротивления току (лампа справа). Это позволяет очень легко применять закон Ома. Если нам известны значения любых двух из трех величин (напряжение, ток и сопротивление) в этой цепи, мы можем использовать закон Ома для определения третьей.

          В этом первом примере мы рассчитаем количество тока (I) в цепи, учитывая значения напряжения (E) и сопротивления (R):

           

           

          Какова сила тока (I) в этой цепи?

           

           

          Во втором примере мы рассчитаем величину сопротивления (R) в цепи при заданных значениях напряжения (E) и тока (I): величина сопротивления (R) лампы?

           

           

          В последнем примере мы рассчитаем величину напряжения, выдаваемого батареей, при заданных значениях тока (I) и сопротивления (R):

           

           

          Какое напряжение обеспечивает батарея?

           

          Метод треугольника закона Ома

          Закон Ома — очень простой и полезный инструмент для анализа электрических цепей. Он так часто используется при изучении электричества и электроники, что серьезный студент должен запомнить его. Для тех, кто еще не освоился с алгеброй, есть хитрость, позволяющая запомнить, как решать любую одну величину, зная две другие.

          Сначала расположите буквы E, I и R в виде треугольника, как показано ниже:

           

           

          Если вы знаете E и I и хотите определить R, просто удалите R с картинки и посмотрите, что осталось :

           

           

          Если вы знаете E и R и хотите определить I, исключите I и посмотрите, что осталось:

           

          , и я хочу определить

           E, уберите E и посмотрите, что осталось:

           

           

          В конце концов, чтобы серьезно изучать электричество и электронику, вам придется хорошо знать алгебру, но этот совет может облегчить запоминание ваших первых расчетов. Если вы хорошо разбираетесь в алгебре, все, что вам нужно сделать, это запомнить E=IR и вывести из нее две другие формулы, когда они вам понадобятся!

           

          ОБЗОР:

          • Напряжение измеряется в вольтах , обозначаемых буквами «Е» или «В».
          • Ток измеряется в амперах , что обозначается буквой «I».
          • Сопротивление измеряется в Ом , обозначается буквой «R».
          • Закон Ома: E = IR ; я = Э/Р; Р = Э/И

          СВЯЗАННЫЕ РАБОЧИЕ ТАБЛИЦЫ:

          • Рабочий лист по закону Ома
          • Рабочий лист юридической практики Ома с ответами

          Попробуйте наш калькулятор закона Ома в разделе «Инструменты».

          Закон Ома и электрическая энергия

          Для анализа цепей постоянного тока, как правило, мы используем различные методы, такие как закон Ома, сетевые теоремы и другие инструменты упрощения схемы. Анализ цепи постоянного тока выполняется в основном для определения неизвестных величин, таких как напряжение, ток, сопротивление и мощность, которые связаны с одним или несколькими элементами электронной схемы. В качестве основного закона упрощения схем закон Ома определяет линейную зависимость между напряжением, током и сопротивлением. Расскажите подробнее об этом законе Ома.

          Схема

          Закон Ома

          Это основной, главный и важный закон, который исследует взаимосвязь между напряжением, током и сопротивлением в электрической цепи. В нем говорится, что при постоянной температуре ток, протекающий по цепи, прямо пропорционален напряжению или разности потенциалов в этой цепи.

          В алгебраической форме V∝ I

          V = IR

          Где

          I — ток, протекающий по цепи, измеряется в амперах

          В — напряжение, приложенное к цепи, измеряется в вольтах.

          И R — константа пропорциональности, называемая сопротивлением, которое измеряется в омах.

          Это сопротивление также указывается в килоомах, мегаомах и т. д.

          Следовательно, закон Ома гласит, что ток в цепи прямо пропорционален напряжению и обратно пропорционален сопротивлению в этой цепи. Закон Ома можно применить как к отдельным частям, так и ко всей цепи.

          Математически, Ток, I = V/R

          Напряжение, В = IR

          Сопротивление, R = V/I

          Вернуться к началу

          Треугольник закона Ома

          Ниже показано соотношение между различными величинами в законе Ома , называется треугольником закона Ом. Это простой метод описания, а также простой для запоминания соотношения между напряжением, током и сопротивлением.

          На рисунке выше показан треугольник закона Ома, где отдельные параметры, такие как напряжение, ток и сопротивление, и их формулы представлены из основного уравнения закона Ома. На приведенном выше рисунке один параметр вычисляется из оставшихся двух параметров. Таким образом, можно сделать вывод, что при высоком сопротивлении ток будет низким, а ток будет высоким, когда сопротивление низкое при любом приложенном напряжении.

          Вернуться к началу

          Электроэнергия

          Электроэнергия показывает скорость, с которой энергия передается по цепи. Электрическая мощность измеряется в ваттах. Эта мощность потребляется, когда напряжение вызывает протекание тока в цепи.

          Таким образом, электрическая мощность является произведением напряжения и силы тока.

          Математически, P = VI

          По закону Ома, V = IR и I = V/R

          Подстановка в уравнение мощности

          P = I 2 R

           P = V 2 / R

          Следовательно, электрическая мощность, P = VI или I 2 R или V 2 / R

          Это три основные формулы для определения электрической мощности в цепи. Таким образом, мощность может быть рассчитана, когда известна любая из двух величин.

          Вернуться к началу

          Треугольник мощности

          Подобно треугольнику закона сопротивления, на рисунке ниже показан треугольник мощности, показывающий соотношение между мощностью, напряжением и током. Уравнения отдельных параметров легко запоминаются по этому рисунку. Округлите и скройте параметр, который необходимо измерить, а положение оставшихся двух параметров дает уравнение для поиска скрытого или округленного параметра, как показано на рисунке ниже.

           

          Вернуться к началу

          Круговая диаграмма закона Ома

          В дополнение к двум вышеупомянутым концепциям существует еще один метод определения параметров цепи с использованием закона Ома, который представляет собой круговую диаграмму закона Ома. Используя круговую диаграмму закона Ома, можно легко запомнить все уравнения для нахождения напряжения, тока, сопротивления и мощности, которые необходимы для упрощения электрических цепей, которые могут быть простыми или сложными.

          На приведенном выше рисунке показана круговая диаграмма, показывающая взаимосвязь между мощностью, напряжением, током и сопротивлением. Эта диаграмма разделена на четыре блока для мощности, напряжения, сопротивления и тока. Каждый блок состоит из трех формул с двумя известными значениями для каждой формулы. Из диаграммы для нахождения каждого параметра в цепи мы можем использовать любую из трех доступных формул.

          Вернуться к началу

          Графическое представление закона Ома

          Для лучшего понимания этой концепции ниже приведена экспериментальная установка, в которой регулируемый источник напряжения с шестью ячейками (каждая по 2 В) подключен к нагрузочному резистору через переключатель напряжения. выключатель. Измерительные приборы, такие как вольтметр и амперметр, также подключены к цепи для измерения напряжения и тока в цепи.

          Регулируемый источник напряжения с нагрузочным резистором

          Сначала подключите резистор 10 Ом и установите переключатель в положение «1». Тогда амперметр показывает 0,2 А, а вольтметр показывает 2 В, потому что I = V/R, т. е. I = 2/10 = 0,2 А. Затем измените положение селекторного переключателя на вторую ячейку, чтобы подать 4 В на нагрузку и запишите показания амперметра. По мере того, как селектор будет постепенно изменяться от первого положения к последнему, мы получим текущие значения, такие как 0,2, 0,4, 0,6, 0,8, 1, 1,2 для значений напряжения 2, 4, 6, 8, 10 и 12 соответственно.

          Аналогичным образом поместите резистор 20 Ом вместо резистора 10 Ом и выполните ту же процедуру, что и выше. Мы получим значения тока 0,1, 0,2, 0,3, 0,4, 0,5, 0,6 для значений напряжения 2, 4, 6, 8, 10 и 12В соответственно. Постройте график этих значений, как показано ниже.

          Графическое представление закона Ома

          На приведенном выше графике при заданном напряжении ток меньше, чем больше сопротивление. Рассмотрим случай приложенного напряжения 12 В, где значение тока составляет 1,2 А при сопротивлении 10 Ом и 0,6 Ом при сопротивлении 20 Ом. Точно так же для одного и того же тока напряжение тем больше, чем больше сопротивление. Из приведенных выше результатов следует, что отношение напряжения к току постоянно, когда сопротивление постоянно. Следовательно, зависимость между напряжением и током является линейной, и наклон этой линейной кривой становится тем круче, чем больше сопротивление.

          Вернуться к началу

          Пример закона Ома

          Рассмотрим приведенную ниже схему, в которой батарея на 6 В подключена к нагрузке 6 Ом. Амперметр и вольтметры подключены к цепи для измерения тока и напряжения практически. Но используя закон Ома, мы можем найти ток и мощность следующим образом.

          Из закона Ома

          В = IR

          I = V/R

          I = 6/6

          I = 1 ампер0002 P = 6 Вт

          Но практически амперметр не показывает точное значение из-за внутреннего сопротивления батареи. Включив внутреннее сопротивление батареи (предположим, что батарея имеет внутреннее сопротивление 1 Ом), текущее значение рассчитывается следующим образом.

          Общее сопротивление цепи 6 +1 = 7 Ом.

          Ток, I = V/R

          I = 6/7

          I = 0,85 А

          Вернуться к началу

          Цепь фар автомобиля

          На приведенном ниже рисунке показана цепь фар автомобиля, за исключением цепи управления. Применяя закон Ома, мы можем узнать ток, протекающий через каждый источник света. Как правило, каждый фонарь подключается параллельно к аккумулятору, что позволяет другим фонарям светиться, даже если кто-то поврежден. К этим параллельным лампам подводится батарея 12 В, где лампы имеют сопротивление 2,4 каждая (считается в данном случае).

          Общее сопротивление цепи равно R = R1x R2/(R1 + R2), так как они соединены параллельно.

          R = 5,76/ 4,8 = 1,2

          Тогда ток, протекающий по цепи, равен I = V/R

          I = 12/ 1,2

          I = 10 А.

          Ток, протекающий через отдельную лампу, составляет I1 = I2 = 5 А (из-за одинаковых сопротивлений)

          Вернуться к началу

          Закон Ома для цепей переменного тока

          В общем, закон Ома также можно применять к цепям переменного тока. Если нагрузка индуктивная или емкостная, то также учитывается реактивное сопротивление нагрузки. Следовательно, с некоторыми изменениями закона Ома, учитывающими влияние реактивного сопротивления, его можно применять к цепям переменного тока. Из-за индуктивности и емкости в переменном токе будет значительный фазовый угол между напряжением и током. А также сопротивление переменному току называется импедансом и обозначается как Z.

          Таким образом, закон сопротивления для цепей переменного тока записывается как

          E = IZ

          I = E/Z

          Z = E/I

          Z — импеданс.

          Все параметры в приведенном выше уравнении представлены в комплексной форме, которая включает фазовый угол. Подобно круговой диаграмме цепи постоянного тока, круговая диаграмма закона Ома для цепи переменного тока приведена ниже.

           

          Вернуться к началу

          Пример закона Ома (цепи переменного тока)

          Рассмотрим приведенную ниже схему, в которой нагрузка переменного тока (сочетание резистивной и индуктивной) подключена к источнику переменного тока 10 В, 60 Гц. Нагрузка имеет сопротивление 5 Ом и индуктивность 10 мГн.

          Тогда значение импеданса нагрузки Z = R + jX L

          Z = 5 + j (2∏×f × L)

          Z = 5+ j (2×3,14× 60×10 × 10-3)

          Z = 5 + j3,76 Ом или 6,26 Ом при фазовом угле -37,016

          Ток, протекающий по цепи, равен

          I = V/Z

          = 10/ (5+ j3,76)

          = 1,597 А при фазовом угле -37,016

          Вернуться к началу

          A, E, I, V, R и Ом ( И P и W) – rAVe [PUBS]

          Том Кер
          Almo Pro A/V

          Alphabet Soup and Math. Не твои любимые, я знаю. Я не получил всего этого сразу, но я поделюсь некоторыми вещами, которые мне очень помогли.

          Сначала разберем эти буквы. У цепей есть свойства, и мы используем значения для количественной оценки этих свойств. Существуют разные символы, используемые в зависимости от того, говорим ли мы о свойствах или значениях, и это сбивает с толку, когда мы смешиваем два символа взаимозаменяемо (как это часто бывает). Один инженер, которого я уважаю, однажды сказал: «Вы никогда не используете символ для количества в формуле, когда на самом деле вычисляете переменную». Кто-то может счесть это более тонким моментом, но он прав. Когда мы работаем с формулами, мы используем переменные. Нам нужно использовать правильный символ, представляющий эту переменную.

          Переменные

          Электродвижущая сила или ЭДС: Сила, вызывающая протекание тока. Это разница в заряде или потенциале между двумя точками, и именно эта разница толкает электроны из точки с высоким потенциалом в точку с низким потенциалом. Символ ЭДС — « E », и мы измеряем эту силу в вольтах, используя символ « V ». Вы также увидите « мВ » при использовании милливольт и « кВ » для киловольт.

          Интенсивность: Количество тока, протекающего в секунду через точку на проводнике. Обозначается интенсивностью « I », и мы измеряем ее в Амперах или Амперах для краткости. Символ для ампер использует « A », и вы часто будете видеть « мА » для миллиампер.

          Сопротивление: Противопоставление потоку тока и символу « R. » Каждая часть цепи оказывает сопротивление, даже сам провод. Мы измеряем это в омах и используем символ омега, Ом . Вы часто будете видеть что-то вроде « 2k Ω » для обозначения 2000 Ом.

          Популярная иллюстрация, которую я нашел полезной для понимания взаимосвязи между напряжением, током и сопротивлением, следующая:

          Импеданс: Цепи постоянного тока (постоянного тока) имеют сопротивление. Цепи переменного тока (переменного тока) имеют сопротивление, а также емкостное и индуктивное сопротивление, которые препятствуют изменениям напряжения и тока в форме волны переменного тока. Величина реактивного сопротивления зависит от частоты, и хотя мы иногда заменяем Z вместо R в формулах, импеданс на самом деле не рассчитываем; мы не можем измерить импеданс с помощью обычного мультиметра. Для импеданса используется символ « Z, », и, как и сопротивление, мы измеряем его в омах и используем символ омега, Ω .

          Мощность: Относится к закону Ома, относится к количеству работы, выполненной в цепи. Мы используем символ « P».  Мы измеряем это в ваттах, используя символ « Вт ». Вы также увидите « мВт “для милливатт или “ кВт ” для киловатт. Побочным продуктом этой работы является тепло, и это может стать проблемой, если мы не сможем правильно управлять теплом в стойке или другом корпусе оборудования.

          Формула закона Ома

          Итак, как мне запомнить формулу закона Ома? «Основы электричества» Van Valkenburgh, Nooger and Neville, Inc. Первоначально она была написана еще в 1954 году и познакомила меня с «Волшебным треугольником»:

          E , I и Р . Закон Ома. Как мне использовать этот волшебный треугольник? Наведите палец на переменную, которую вы хотите найти, и посмотрите, что осталось. Хотите решить для E ? Проведите пальцем по E ; то, что осталось, выглядит как I раз (x) R . Решение для I ? Это выглядит как E на R или E разделить на (÷) на R. То же самое с решением для R. Это E разделить на (÷) I.

          Какое практическое применение? Допустим, у меня есть устройство, которому требуется 12 В постоянного тока, а блок питания, поставляемый с устройством, рассчитан на 13,5 В постоянного тока и обеспечивает 0,5 А. Чтобы все выглядело аккуратно, я хочу удалить блок питания на расстоянии 20 метров и буду использовать провод 24 AWG. Напомним, что провод имеет сопротивление, и мне нужно рассчитать падение напряжения на проводе. Падение напряжения просто относится к потере электрического давления, когда ток протекает через сопротивление. Цель состоит в том, чтобы увидеть, будут ли у меня все еще необходимые 12 В на конце 65-футового провода.

          Что я знаю? Давайте узнаем :

          Блок питания обеспечивает ток 0,5 А, а обычный многожильный медный провод марки 24 AWG имеет сопротивление 0,026 Ом на фут. Хотя источник питания находится на расстоянии 65 футов, на самом деле это 130 футов провода. Почему? Туда и обратно, плюс и минус цепи — это два провода, а не один. 0,026 на фут x 130 футов = 3,38 Ом общего сопротивления на этом участке провода.

          Давайте найдем Е, падение напряжения на проводе.

          • Э = Я х Р
          • E = 0,5 А x 3,38 Ом
          • Е = 1,69 В

          Я начал с 13,5 В от источника питания за вычетом 1,69 В, которые я теряю на 130-футовом проводе 24 AWG. 13,5 минус (-) 1,69 оставляет мне только 11,81 В для устройства, которому требуется 12 В. Упс. Мой выбор состоит в том, чтобы либо укоротить провод, либо использовать провод AWG большего размера.

          Если я использую провод 22 AWG с сопротивлением 0,0165 Ом на фут (0,0165 на фут x 130 футов = 2,145 Ом):

          • E = 0,5 A x 2,145 Ом
          • Е = 1,0725 В

          13,5 В минус (-) падение напряжения на проводе 1,0725 дает мне 12,43 В для устройства. Устройство на 12 В будет счастливо, и жизнь наладится. Звонок в сервис не нужен!

          Сила, связанная с законом Ома, имеет свой собственный «магический треугольник»:

          Кто-то скажет «это так же просто, как PIE » или « P = I x E. ». Точно так же, как треугольник закона Ома, поместите палец над переменной, для которой вы хотите найти:

          • Р = Я х Е
          • Я = П/Э
          • E = P/I

          Мы часто используем это для расчета текущего потребления оборудования. Предположим, что у нас есть два новых 4K-проектора с яркостью 7000 люмен для использования в классе. Я хочу знать, могу ли я подключить оба проектора к одной и той же цепи на 20 А, но мне нужно знать, какой ток будет потреблять каждый проектор. В спецификациях указано, что максимальная потребляемая мощность составляет 1070 Вт. Я положил палец на букву I, и формула выглядит так: I = P / E.

          Что я знаю? Давайте узнаем:

          • P = 1070 Вт из спецификации.
          • E = 120 В — электросеть здесь, в США
          • Я = П/Э
          • I = 1070 Вт / 120 В
          • I = 8,92 ампер

          8,92 x 2 проектора = 17,84 А. Несмотря на то, что это может быть цепь на 20 А, NEC сообщает мне, что я не могу подавать непрерывную нагрузку более 16 А на цепь на 20 А. Поскольку проекторы, по сути, представляют собой большие лампочки, я бы считал их непрерывными нагрузками, и почти 18 А превышают допуск 16 А. Мне нужно поставить проекторы на отдельные цепи 20 А.

          Это работает даже для фенов (и тепловых пушек, которые мы используем с термоусадочными трубками). Допустим, я покупаю новый фен мощностью 1200 Вт. Каков текущий розыгрыш?

          • И = П/Э
          • I = 1200 Вт / 120 В
          • I = 10 А

          Моя новая тепловая пушка мощностью 1740 Вт потребляет 14,5 А

          • I = P / E
          • I = 1740 Вт / 120 В
          • I = 14,5

          Таким образом, если вы подключите второй фен или тепловую пушку к той же цепи и попытаетесь использовать их одновременно, результатом будет срабатывание автоматического выключателя.

          Колесо формул

          Иметь полное колесо формул, содержащее формулы для решения всех четырех свойств, безусловно, удобно:

          Но если у вас нет колеса, треугольники EIR и PIE легко запомнить и использовать. Вот несколько полезных ссылок:

          • Fujitaki, K. (2009). «Руководство манги по электричеству». Без Крахмального Пресса.
          • Ван Валкенбург, Нугер, Невилл. (1992). «Основное электричество». Срочные публикации.
          • Холт, М. (2004). «Основы теории электричества». Майк Холт Энтерпрайзис, Инк.
          • Легран А.В. «Оптимизированное распределение питания и заземление для аудио-, видео- и электронных систем»

          Надеюсь, это помогло вам лучше понять закон и силу Ома, а также то, как все это взаимосвязано.

          Как понять закон Ома – A Shady Review

          Закон Ома

          Если вы уверены в своих знаниях о безопасности аккумуляторов для вейпов, следующим соображением будет использование какого-либо калькулятора, чтобы убедиться, что ваши сборки катушек находятся в пределах этого безопасного пределы вашей батареи и многое другое, чтобы вы могли настроить свои катушки, чтобы получить желаемый опыт вейпинга. Существует множество калькуляторов закона Ома и такие сайты, как Steam Engine, которые сделают всю тяжелую работу за вас.

          Если вас это устраивает, и вы хотите оставаться в блаженном неведении относительно того, что на самом деле стоит за расчетами, хорошо для вас. Пока вы знаете, как применять результаты, вы проживете долгую, счастливую и безопасную жизнь вейпинга. Но если вы хотите увидеть внутреннюю работу этих калькуляторов, читайте дальше.

          В законе Ома нет ничего мистического или магического. Это несколько формул, обычно изображенных внутри треугольника, и любой может легко выучить и использовать формулы с любым обычным калькулятором.

          Цель здесь — показать вам формулы, лежащие в основе закона Ома, и, надеюсь, дать вам представление о взаимосвязи между различными элементами в базовой электронной схеме, связанной с вейпингом.

          Треугольник

          Внутри треугольника вы можете увидеть три основных элемента в любой электрической цепи, представленные буквами V, I и R. Я бы озвучил треугольник как «V больше I, умноженное на R» с «раз ” является умножением. Сложнее всего будет запомнить, что обозначают буквы, а это легко:

          В = Напряжение (напряжение вашей батареи)

          I = Ток (сила тока, потребляемая вашей катушкой)

          R = Сопротивление (сопротивление вашей катушки в омах)

          Итак, как мы используем закон Ома треугольник? Опять же, просто — треугольник наглядно изображает взаимосвязь между напряжением, током и сопротивлением. В следующих примерах мы рассмотрим, как использовать треугольник и формулы, чтобы помочь вам построить катушки, ориентированные на ток и мощность, которые вы хотите.

          Расчетный ток

          Если вы хотите определить потребляемый ток через сопротивление (ваша катушка), используйте следующую формулу:

          I = V ÷ R (или I = V/R)

          Как мы к этому пришли? Посмотрите на треугольник, и вы увидите, что для его решения для тока (I) вы должны разделить напряжение (V) на сопротивление (R).

          Давайте применим формулу к реальному примеру. Если вы используете механический мод, со свежезаряженной батареей у вас теоретически есть 4,2 В для питания катушки. Если ваша катушка 0,5 Ом, теперь у вас есть все необходимое для определения тока в амперах:

          I = 4,2 В ÷ 0,5 Ом (или 4,2/0,5)

          I = 8,4 А

          Как видите, с вашей катушкой 0,5 Ом и свежезаряженной батареей на 4,2 В результирующее максимальное потребление тока будет 8,4 ампер. Если ваша батарея имеет ограничение в 10 ампер, вы значительно ниже предела. Не забывайте, что использование двойного механического мода в последовательной конфигурации удвоит потребление усилителя на батарею, и вам придется создавать катушки с вдвое более высоким сопротивлением, чтобы быть в безопасности. Также обратите внимание, что по мере разрядки батареи ток также будет уменьшаться. Например, когда напряжение батареи достигает 3,7 В при той же нагрузке, ток упадет до 7,4 А (3,7 В/0,5 Ом)9.0003

          Расчет мощности (в ваттах)

          Следующая вещь, которую вы, возможно, захотите узнать, это мощность, вырабатываемая катушкой, или мощность в ваттах. В треугольнике это не показано, но формула проста. Просто умножьте ток в вашей цепи на приложенное напряжение:

          P = V x I

          В нашем исходном примере формула будет выглядеть так:

          P = 4,2 В x 8,4 А

          P = 35,3 Вт

          Так что катушка 0,5 Ом при полностью заряженном аккумуляторе на 4,2 вольта будет тянуть максимум 8,4 ампера и выдавать 35,3 ватта. Вы можете видеть, что по мере увеличения сопротивления вашей катушки ток и мощность падают.

          Расчет сопротивления

          Вторая формула закона Ома, которая может нам пригодиться, это расчет сопротивления. Допустим, у вас есть батарея с ограничением по току 10 ампер, и вы хотите определить наименьшее сопротивление катушки, которое вы можете безопасно использовать, не превышая CDR батареи.

          Для расчета используйте следующую формулу:

          R = V ÷ I

          Поскольку вы знаете, что CDR батареи составляет 10 ампер, вы можете ориентироваться на 9 ампер в своих расчетах, чтобы получить 1 ампер запаса. . Вы также знаете, что ваше максимальное напряжение будет 4,2 вольта на моде с одной батареей. Таким образом, расчет выглядит следующим образом:

          R = 4,2 В ÷ 9 А

          R = 0,47 Ом

          Результат говорит вам, что ваш самый безопасный нижний предел с 10-амперной батареей составляет 0,47 Ом – что бы ни было ниже, вы рискуете превысить предел тока батареи. Конечно, если у вас 25-амперная батарея, ваше низкое сопротивление упадет до 0,17 Ом:

          R = 4,2 В ÷ 25 А

          R = 0,17 Ом

          Расчет напряжения

          Наконец, и, вероятно, не так полезно для нас, используя треугольник, вы можете найти напряжение в цепи, если вы знаете значения двух других переменных.

          Чтобы найти напряжение, когда известны ток и сопротивление, формула выглядит так:

          В = I x R

          Что все это значит?

          Действительно, наиболее полезными для вейперов являются те три формулы, которые рассчитывают ток (I = V ÷ R), мощность (P = V x I) и сопротивление (R = V ÷ I). Это позволит вам определить ток, который будет потреблять ваша катушка, и полученную мощность. По мере увеличения сопротивления ток и мощность будут падать. Если уменьшить сопротивление, ток и мощность увеличатся. Формула сопротивления позволяет рассчитать безопасное низкое сопротивление на основе CDR вашей батареи.

          Это полезная информация, которая поможет вам оставаться в безопасных пределах ваших батарей и настроить количество энергии на вашей катушке, чтобы помочь вам достичь своей собственной нирваны. Есть и другие соображения, такие как время разгона катушки и тепло вашей катушки, которые определяются сечением проволоки и массой. Закон Ома ничего из этого не учитывает, и такой сайт, как Steam Engine, может быть полезен.

          Последний и важный совет: ВСЕГДА предполагайте, что напряжение вашей батареи эквивалентно полностью заряженной батарее: 4,2 вольта для модуля с одной батареей или модуля с параллельными батареями или 8,4 вольта для модуля с двумя последовательностями. Люди будут утверждать, что катушка никогда не увидит фактическое напряжение батареи из-за падения напряжения внутри мода, но, чтобы быть в безопасности, ВСЕГДА используйте полное теоретическое напряжение батареи (при полной зарядке) в своих расчетах. Или, если вы все еще не уверены, что я использую vapecalc.com

          Дэвид Трош

          Главный редактор и автор

          Примеры закона Ома с формулой определения и треугольником

          Закон Ома — это математическая формула, описывающая взаимосвязь между напряжением, током и сопротивлением в электрической цепи.

          Закон назван в честь немецкого физика Георга Симона Ома.

          Тем не менее, читатель должен учитывать, что не все проводники работают по закону Ома, так как некоторые проводники называются неомическими, значение сопротивления которых меняется в зависимости от приложенного напряжения.

          Закон Ома Определение

          Закон Ома гласит, что «сила постоянного тока в электрическом проводнике прямо пропорциональна разности потенциалов и обратно пропорциональна удельному сопротивлению проводника».

          Формула закона Ома

          С математической точки зрения закон Ома применяется по уравнению:

           

          Где:

          • «R» — электрическое сопротивление. Под сопротивлением мы подразумеваем препятствие, которое ток встречает на своем пути, например, лампочку. Чем выше сопротивление, тем труднее току будет пройти через секцию. Единицей измерения сопротивления является ом, обозначаемый греческой буквой омега (Ом).

          • «I» — это сила электрического тока (сила тока), проходящего через поперечное сечение проводника, испускаемого в амперах (А).

          • «В» — напряжение; это электрическая энергия, с которой батарея или другое электрическое устройство гонит электроны по цепи. Мощность более значительна, когда напряжение выше; поэтому электроны текут быстрее, увеличивая электрический ток. Напряжение или напряжение измеряется в вольтах (В).

          Как мы видели в определении закона Ома, сила тока прямо пропорциональна приложенному напряжению и обратно пропорциональна сопротивлению. Если напряжение падает, сила тока падает пропорционально.

          Для напряжения, с другой стороны, чем оно выше, тем большую силу притяжения оно создает для перемещения зарядов, поэтому при одном и том же значении сопротивления оно будет прямо пропорционально току.

          Закон Ома для переменного тока (AC)

          Для анализа цепей переменного тока сопротивление (R) заменяется импедансом (Z), который учитывает фазовые сдвиги между напряжением и силой тока, а также влияние электромагнитных полей, создаваемых в цепи. компоненты электронной схемы. Но самое нормальное в базовой электронике — анализировать цепи на постоянном токе или применять упрощения, позволяющие изучать их такими, какими они были.

          Закон Ома и электрическая мощность

          Иногда формула для электрической мощности используется путем применения закона Ома.

          Формула мощности выглядит следующим образом (в ваттах):

          P = V·I, (мощность = напряжение x ток)

          и его варианты: V = P/I и I = P/V,

          Два варианта можно заменить в формуле закона Ома. Например, если мы начнем с формулы для расчета напряжения и подставим, мы получим следующую формулу:

          V = (P / V) R

          Изолируя мощность, мы имеем следующую форму:

          V² · R = P

          Что такое треугольник закона Ома?

          Треугольник закона Ома — это уловка для запоминания формулы.

          Чтобы получить формулу треугольника, мы должны покрыть переменную, которую мы хотим получить, руками. Если оставшиеся элементы находятся поверх других, они разделяются. Если они остаются в строке, мы должны умножить значения.

          Возможны три комбинации:

          • I = V / R

          • V = I·R

          • R = V / I

          гидравлический пример:

          Давайте представим шланг, подсоединенный к баку пожарной машины.

          В этом примере элементами, эквивалентными электрической цепи, являются следующие:

          • Напряжение (В). Напряжение равно мощности водяного насоса грузовика. Водяной насос «подает напряжение» на цепь, как если бы это была батарея в электрической цепи.

          • Сопротивление (R). Сопротивление электрической цепи эквивалентно сопротивлению шланга (площадь окружности). Если шланг большого диаметра, вода будет течь легче, чем если шланг маленького диаметра. В примере с электрической цепью понятие сопротивления эквивалентно, но выражается в Омах (Ом).

          • Сила тока (I): В нашем примере сила тока — это количество молекул воды, протекающих через секцию шланга в единицу времени. В электрической цепи эквивалентом молекул являются электрические заряды.

          Из этого сравнения легко понять, что если мы увеличим производительность насоса (напряжение), расход воды (сила тока) увеличится. Точно так же, если мы используем шланг меньшего размера (большее сопротивление), поток также уменьшится.

          Примеры практических задач по закону Ома

          Перейдем к решению некоторых численных задач по закону Ома:

          Пример 1 задачи по закону Ома

          Найдите силу тока, питающего игрушечную стиральную машину с сопротивлением 10 Ом и работает от аккумулятора с разностью потенциалов 30 В.

          Решение: Для решения этой задачи достаточно вернуться к данным задачи, которые в данном случае будут сопротивлением 10 Ом, и напряжением 30 Вольт, что бы мы имели.

          R=10 Ом

          В=10 В

          I=?

          Задача требует от нас тока, поэтому мы должны применить закон Ома, чтобы найти его.

          I=V·R=30v10Ω=3A

          Окончательный вывод: нам нужно три ампера для питания игрушечной стиральной машины. Как видим, сила тока пропорциональна напряжению.

          Пример 2 задачи по закону Ома

          Найдите напряжение между двумя точками цепи, через которые проходит ток силой 4 ампера и сопротивление которого равно 10 Ом.

          Решение: Как и в предыдущем примере, нам нужно вернуться к нашим данным, которые в данном случае будут четырьмя амперами, пересекающими железную цепь, и резистором 10 Ом, поэтому:

          I=4A

          R=10 Ом

          В=?

          В этом случае наша формула будет такой же, только теперь мы ее очистим.

    • Оставить комментарий